• Shuffle
    Toggle On
    Toggle Off
  • Alphabetize
    Toggle On
    Toggle Off
  • Front First
    Toggle On
    Toggle Off
  • Both Sides
    Toggle On
    Toggle Off
  • Read
    Toggle On
    Toggle Off
Reading...
Front

Card Range To Study

through

image

Play button

image

Play button

image

Progress

1/127

Click to flip

Use LEFT and RIGHT arrow keys to navigate between flashcards;

Use UP and DOWN arrow keys to flip the card;

H to show hint;

A reads text to speech;

127 Cards in this Set

  • Front
  • Back

tips: memorize malice crimes and specific intent crimes - all others are general intent

note that students often miss the element of intent

Exam Tip 1: When you read your exam question, ask yourself what is the requisite intent? Specific intent, malice, or general intent. Remember the defenses that apply only to ______________ intent crimes. Example 1: I own a black 2002 Chevrolet. You own a brand-new, red BMW. After 2 days of bar exam exhaustion, I jump in your shiny red BMW. When the police pull me over, I explain that I honestly believed that I jumped in my own car. This may be an unreasonable belief, but if it is honestly held, it is a defense to the specific intent crime of theft.

specific

two defenses only available with specific intent crimes:

voluntary intoxication any mistake of fact, even an unreasonable one

Arrest, search and seizure have lots of questions

- note: privileges against self-incrimination and right to counsel - note: always one question on double jeapardy

tip: when you see a crime, look for the intent an "attempted" crime is always a __________ intent crime, so are all property offenses

specific

A man planned to rob a city bank. Waiting until he saw the bank’s security guard leave the building to take a walk around the perimeter, the man entered the bank and walked up to a teller with his hand pointed in his pocket, as if he had a gun. The teller had her back to the man and did not see him at all. Before the teller could turn around, the security guard re-entered the building. Seeing the guard, the man took his hand out of his pocket and ran out of the building. The man was arrested and charged with attempted robbery of the bank. Should the man be found guilty?

A. Yes, whether or not he abandoned his plan. B. Yes, because robbery is a specific intent crime. C. No, because he successfully withdrew from the robbery. D. No, because he used no actual force on the teller.

Answer choice A is correct. An attempt requires a specific intent to commit a criminal act coupled with a substantial step taken toward the commission of the intended crime. At common law, once the defendant has taken a substantial step toward the commission of the offense, the defendant may not legally abandon the attempt to commit the crime because of a change of heart. Upon the completion of a substantial step, the crime of attempt is completed; there can be no abandonment or withdrawal. The facts here indicate that the man planned and intended to rob the bank, so the specific intent element is satisfied. He entered the building and pretended to have a gun in his pocket. This constitutes a substantial step toward the commission of the intended crime

Answer choice B is incorrect because, while a true statement, it has no bearing on whether the man should be found guilty of attempted robbery; the crime of attempt can be found even when the underlying crime is a general intent crime. Answer choice C is incorrect, as withdrawal would not constitute a common-law defense. Note that even in a jurisdiction that recognizes abandonment or withdrawal as a defense to attempt, the man’s abandonment was not voluntary and therefore does not constitute a defense; the abandonment was motivated by a desire to avoid detection by the security guard. Answer choice D is incorrect, as it is irrelevant that the man did not use actual force on the teller. As noted above, attempt merely requires specific intent to commit the criminal act along with a substantial step toward the commission of the intended crime. While robbery requires the use of force or intimidation, attempted robbery does not.

A woman was envious of her sister’s engagement ring, which had a diamond much larger than the one in the woman’s own engagement ring. After a dinner at the sister’s house, the woman and her sister each placed their rings in a jewelry dish next to the sink while they washed dishes. When the dishes were finished, the woman took her sister’s ring, placed it on her finger, and left to go home. When the sister noticed that her ring was gone, she was furious and demanded that the woman return the ring immediately, which the woman did. The woman honestly claims that she mistakenly thought the ring was hers, but the sister has nonetheless asked the local prosecutor to press charges for theft. Is the woman likely to be convicted of larceny?

A. No, because the woman’s mistake was honest, even if it was unreasonable. B. No, because she returned the ring as soon as her sister requested she do so. C. Yes, because mistake of fact is not a valid defense for specific intent crimes. D. Yes, because the woman’s mistake must be both honest and reasonable to support a mistake of fact defense.

Answer choice A is correct. Mistake of fact may negate criminal intent for both specific and general intent crimes if the mistake is an “honest mistake.” For specific intent crimes, a mistake of fact is a valid defense even if the mistake is unreasonable. Because theft crimes are specific intent crimes, the woman may assert a mistake of fact defense so long as the mistake was honest, even if it was unreasonable. Answer choice B is incorrect because, if the elements of larceny are satisfied, the larceny is complete as soon as the property is carried away. There is no defense of restoration if the defendant later has a change of heart and restores the property to its rightful owner. Answer choice C is incorrect because the mistake of fact defense applies to both specific and general intent crimes.

Answer choice D is incorrect because it describes the mistake of fact defense as it applies to general intent crimes. Theft crimes are specific intent crimes, however, and thus a mistake of fact need not be reasonable so long as it is honest.

Plagued by neighborhood youths who had been stealing lawn furniture from his backyard, a homeowner remained awake nightly watching for them. One evening the homeowner heard noises in his back yard. He yelled out, warning intruders to leave. Receiving no answer, he fired a shotgun filled with nonlethal buckshot into bushes along his back fence where he believed the intruders might be hiding. A six-year old child was hiding in the bushes and was struck in the eye by some of the pellets, causing loss of sight.

If the homeowner is charged with second-degree assault, which is defined in the jurisdiction as "maliciously causing serious physical injury to another," he is: A. not guilty, because the child was trespassing and he was using what he believed was nondeadly force. B. not guilty, because he did not intend to kill or to cause serious physical injury. C. guilty, because he recklessly caused serious physical injury. D. guilty, because there is no privilege to use force against a person who is too young to be criminally responsible.

C

Answer choice C is correct. Malice may be inferred from a reckless disregard to an obviously high risk of a harmful result. Here, the homeowner recklessly fired into the dark at an unseen object to protect his furniture, resulting in foreseeable harm to the child. Answer choice A is incorrect because, even if nondeadly, this level of force was unreasonable and was disproportionate to the threat to the furniture. Answer choice B is incorrect because the recklessness of firing the gun into the darkness was sufficient to establish intent. Answer choice D is incorrect because there is no duty to refrain from using force against a young child when that child is engaged in a dangerous act, and because the homeowner's actions were reckless regardless of the age of the person who was hurt.

A bartender is charged with the statutory offense of "knowingly violating a regulation of the State Alcoholic Beverage Control Board" and specifically that he knowingly violated regulation number 345-90 issued by the State Alcoholic Beverage Control Board. That regulation prohibits the sale of alcoholic beverages to any person under the age of 18 and also prohibits the sale of any alcoholic beverage to a person over the age of 17 and under the age of 22 without the presentation of such person's driver's license or other identification showing the age of the purchaser to be 18 or older. The evidence showed that the bartender worked in a tavern and sold a bottle of beer to a person who was 17 years old and that the bartender did not ask for or see the purchaser's driver's license or any other identification.

Which of the following, if found by the jury, would be of the most help to the bartender? A. The purchaser had a driver's license that falsely showed his age to be 21. B. The bartender had never been told he was supposed to check identification of persons over 17 and under 22 before selling them alcohol. C. The bartender did not know that the regulations classified beer as an alcoholic beverage. D. The bartender mistakenly believed the purchaser to be 24 years old.

D note: this is a specific intent crime - ignorance of the law is not defense, but a mistake of fact is a defense (even if the mistake of fact is unreasonable)

Answer choice D is correct. Because the bartender is charged with knowingly violating the statute, information that the bartender did not know the individual was underage (a mistake of fact) goes to negate the specific intent required by the statute. Answer choice A is incorrect because there is no indication that the bartender ever actually saw that license. Answer choices B and C are incorrect because ignorance of the law is no defense, and is not what is intended by "knowingly violating" the statute.

A police officer, acting on an anonymous tip that a farmer was illegally growing opium poppies in his fields, entered the farm without permission of the farmer by climbing over a locked gate and despite the presence of several "No trespassing" signs. Searching the property, the officer found the poppies growing in a field that was surrounded by woods and consequently not visible from any other privately owned or public property. In addition, the farmer had deployed camouflage netting to forestall detection of the poppies from the air. Based on her observations, the police officer arrested the farmer and seized the poppies. Prior to the farmer's trial for illegally growing opium poppies, the farmer filed a motion to suppress the poppies on the basis that they were seized in violation of his constitutional rights.

Should the poppies be suppressed? A. No, because the Fourth Amendment protection against unreasonable searches and seizures does not extend to open fields. B. No, because the police office had probable cause to search the farmer's property without a warrant. C. Yes, because the police officer gained knowledge of the poppies by trespassing on the farmer's property. D. Yes, because the farmer made reasonable efforts to exclude the public from his lands.

A (we could fly over and see what was in that field - open fields doctrine, despite signs and fences)

Answer choice A is correct. By its terms, the Fourth Amendment is limited to "persons, houses, papers and effects;" it does not extend to open fields. Whether a location is an open field does not turn on whether the owner of the field has taken measures to keep it private, but on whether such expectations of privacy are objectively reasonable. For activities conducted outdoors, a person generally has no reasonable expectation of privacy. For this reason, answer choice D is incorrect. Answer choice B is incorrect because an anonymous tip, without more, is not sufficient to constitute probable cause to conduct a search. Answer choice C is incorrect because, although the police officer may be liable in tort to the farmer or even subject to criminal action for trespassing, the officer's trespass does not prevent her search from being constitutional. The law of trespass proscribes intrusions upon land that the Fourth Amendment does not.

A grand jury returned an indictment charging the defendant with bank robbery, and when he could not make bond he was jailed pending trial. He had received Miranda warnings when arrested and had made no statement at that time. The prosecutor arranged to have an informant placed as the defendant's cellmate and instructed the informant to find out about the bank robbery without asking any direct questions about it. The informant, once in the cell, constantly boasted about the crimes that he had committed. Not to be outdone, the defendant finally declared that he had committed the bank robbery with which he was charged. At the defendant's trial, his attorney moved to exclude any testimony from the informant concerning the defendant's boast.

The motion should be: A. granted, because the defendant's privilege against self-incrimination was violated. B. granted, because the defendant's right to counsel was violated. C. denied, because the defendant had received Miranda warnings. D. denied, because the defendant was not interrogated by the informant.

Answer choice B is correct. The Sixth Amendment right to counsel attaches when the state initiates prosecution with an indictment and applies to all critical stages of the prosecution, including custodial police interrogations. Interrogation refers not only to express questioning, but also to any words or actions that the police know or should know are likely to elicit an incriminating response. Placing the informant in the jail cell with the defendant was a police action that was intended and likely to elicit an incriminating response, making it a custodial interrogation that violated the defendant's right to counsel.

Therefore, answer choice D is incorrect. Answer choice A is incorrect because the defendant's statement was voluntary and he had been given Miranda warnings; his Fifth Amendment privilege was not violated. Answer choice C is incorrect because Miranda warnings apply to Fifth Amendment rights, and the issue here is the defendant's Sixth Amendment right to counsel.

(a) The issue is whether a defendant can successfully raise the affirmative defense of justification based on an imminent threat of harm. Self-defense is a justification defense that allows a nonaggressor to use only as much reasonable force as is required to defend against the imminent attack of another. New York permits the use of deadly force in self-defense when the defendant reasonably believes that deadly force is about to be used against him and he cannot retreat with complete safety. Deadly force includes force that is likely to cause serious bodily injury.

Here, defendant Joe’s use of a gun against victims Ron and Sam is deadly force. Mark had threatened Joe that he would have Joe killed by gang members. Therefore, when Ron and Sam ran toward Joe wearing gang insignia, Joe was reasonable in assuming that Ron and Sam were about to use deadly force against him. Further, Joe was in a parking lot near his car. Neither the lot nor the vehicle can be said to have been a place to which Joe could retreat with complete safety from the imminent threat of deadly force. Thus, Joe is likely to be successful in raising the justification defense.

(b)(1) The issue is whether the actions of Ron and Sam constitute the crime of attempted assault. An attempt requires a specific intent to commit a criminal act coupled with conduct that tends to effect the commission of the intended crime. Such conduct may include preparatory acts. Under New York law, the common-law crime of battery is known as assault. Common-law battery is defined as the unlawful application of force to another person that causes bodily harm to that person or constitutes an offensive touching.

Here, Ron and Sam had the specific intent to unlawfully apply force on Joe that would cause him bodily harm. Arguably, their action in running toward Joe was in preparation to unlawfully apply force on Joe that would cause him bodily harm. Thus, a jury could find that Ron and Sam committed the crime of attempted assault.

(b)(2) The issue is whether Ron and Sam have any defenses to the crime of attempted assault. To defend against a crime, a criminal defendant can seek to demonstrate to a jury that the prosecution failed to demonstrate every element of the crime beyond a reasonable doubt. In the alternative, a defendant may raise an affirmative defense whereby the defendant proffers an excuse for having committed such crime and establish that defense by a preponderance of the evidence.

Here, although the defendants clearly had the intent to commit an assault, they may be successful in arguing that running toward Joe was, in and of itself, insufficient to effect the commission of the crime. Under New York law, the requirement that a defendant engage in conduct that tends to effect the commission of the crime means that the defendant’s conduct must come “very near” or “dangerously near” consummation of the crime. Here, the defendants may be successful in convincing a jury that their action of running, in and of itself, was insufficient to come close to harming Joe. (continued)

What is another possible affirmative defense?

Absent being able to convince a jury that their actions were insufficient to make up the crime of attempted assault, the defendants may raise their age as an affirmative defense to the crime. Under New York penal law, persons less than 16 years of age are not criminally responsible for certain crimes. However, there are exceptions to the rule for violent crimes. Juveniles ages 14 or 15 are criminally responsible for attempted assault. Here, Ron and Sam were aged 15, and thus their age alone will not preclude them from being convicted of attempt to commit assault.

(c)(1) The issue is whether Mark’s actions constitute the crime of conspiracy. Under New York law, the crime of conspiracy requires an agreement to commit an unlawful purpose, with the intent to accomplish that purpose, and the performance of an overt act in furtherance of that purpose. The overt act of any conspirator may be attributable to the other coconspirators to establish the offense of conspiracy.

Here, Mark agreed with Ron and Sam that Ron and Sam should harm Joe. Mark intended that Ron and Sam would, in fact, beat up Joe. Ron and Sam saw Joe and ran toward him with the intent to beat him up. Therefore, their actions in furtherance of that objective were sufficient to constitute an overt act that would make Mark responsible for conspiracy, even though Mark did not commit the action himself.

(c)(2) The issue is whether Mark’s actions constitute the crime of solicitation. Solicitation is the enticing, encouraging or advising of another to commit a crime, with the intent that the person commits that crime. In New York, criminal solicitation does not merge with the completed crime unless the solicitation is necessarily incidental to that crime; that is, a person may be charged with both solicitation and the completed crime.

Here, Mark contacted Ron and Sam and encouraged them to beat up Joe. His intent was that they give Joe a severe beating. All the elements of solicitation were met at this point because solicitation does not require that the solicited parties agree to carry out the crime. Therefore, Mark’s conduct is criminal solicitation.

A mugger approached an elderly married couple and held them at gunpoint, demanding money. When the husband resisted, the mugger ran off. A nearby police officer witnessed the incident and raced after the mugger in hot pursuit. The mugger turned and pointed his gun at the officer. The officer shot wildly in self-defense, unexpectedly killing an innocent bystander. The jurisdiction follows the agency theory of felony murder. If the mugger is charged in connection with the death of the bystander, which of the following crimes, listed in descending order of seriousness, is the most serious for which he is likely to be convicted? A. Felony murder. B. Voluntary manslaughter. C. Involuntary manslaughter. D. No crime.

D Answer choice A is incorrect because, under the agency theory of felony murder adopted by a minority of states, a felon is not liable for the death of a bystander caused by a police officer, because the officer is not the felon’s agent. In this case, the death of a bystander does not fall within the felony murder rule because the officer is not an agent of the perpetrator of the felony. (Note that under the majority approach, known as the proximate cause theory, the bystander’s death would fall within the felony murder rule because the death was a direct result of the underlying felony.) Answer choice B is incorrect because voluntary manslaughter is a killing that, except for adequate provocation, would be murder. In this case, had the mugger’s actions met the requirements for murder, there were no extenuating circumstances that would warrant dropping the charge to voluntary manslaughter. Answer choice C is incorrect because involuntary manslaughter is an unintentional homicide comm

A woman learned that her friend was having an affair with the woman’s husband, and she told her sister about this. The sister had always hated the woman’s husband, and together, they planned to murder the husband to collect a $2 million life insurance policy for which the woman was the sole beneficiary. The woman promised to give her sister $500,000 if she murdered the husband and gave her $10,000 up front. The sister purchased a gun, which she planned to use to shoot the woman’s husband. She waited in the parking garage when the husband left his office, but she was unable to pull the trigger. The sister later told the woman that she could not go through with the plan and gave back the $10,000. The jurisdiction recognizes the majority rule regarding conspiracy liability.

Which of the following best describes the sister’s criminal liability? A. The sister is guilty of attempted murder and conspiracy to commit murder. B. The sister is guilty of attempted murder only. C. The sister is guilty of conspiracy to commit murder only. D. The sister is not guilty of any crime.

Answer choice A is correct. Conspiracy is an agreement between two or more persons to accomplish an unlawful purpose with the intent to accomplish that purpose. The majority rule requires an overt act in furtherance of the conspiracy. Under the majority rule, withdrawal is possible between the date of the agreement and the commission of the overt act. Upon completion of the overt act, the conspiracy is formed and withdrawal is no longer possible. In this case, the conspiracy was complete when the sister purchased the gun and laid in wait for the husband, and thus her later actions were insufficient to withdraw from the conspiracy. An attempt requires a specific intent to commit a criminal act coupled with a substantial step taken toward the commission of the intended crime, which fails to be completed. Preparatory acts, such as lying in wait for the victim or possessing materials required for the crime, may constitute a substantial step if they corroborate the defendant’s criminal

purpose. In this case, the sister purchased a gun and then waited in the garage to shoot the husband, and thus completed a substantial step toward murder. Answer choices B is incorrect because the sister is also guilty of conspiracy, as she made the agreement and made an overt act in furtherance of that. Answer choice C is incorrect because the sister took a substantial step in the commission of the crime and is therefore guilty of attempted murder as well. Answer choice D is incorrect because the sister would be guilty of both attempt and conspiracy.

A guest at a party becomes enraged over the host's advances towards the guest's wife. Threatening to burn down the host's home, the guest sets a chair on fire. In addition to consuming the chair, the fire causes extensive smoke damage to the room before it is put out. The guest is charged with common-law arson. Should he be convicted? A. No, because the fire did not damage the home. B. No, because the guest did not enter the home with the intent to set it on fire. C. Yes, because the structure was the host's home. D. Yes, because the guest had ill will towards the host.

Answer choice A is correct. In order for a fire set in a dwelling to be arson, there must be some damage (e.g., charring) to the structure of the dwelling. Smoke damage alone is insufficient. Answer choice B is incorrect because, unlike burglary, arson does not require that defendant enter the structure with the intent to commit a crime. Answer choice C is incorrect because, even though the structure where the fire took place was the dwelling of another, there was no damage to its structure. Answer choice D is incorrect because the guest's ill will towards the host may explain his motive for acting, but is not required to establish malice. Here, the guest's threat coupled with his act would likely be sufficient to establish the necessary malice.

9. (Question ID#3068) A man placed an advertisement on an internet website offering to sell a rare coin in return for $50,000. A woman responded to the advertisement, met the man, inspected the coin, and offered to buy it. The man told her that others had expressed an interest in the coin, but that she could have it if she paid a $15,000 cash deposit within one day, with the balance paid by the end of the week. The woman came back the next day and gave the $15,000 to the man, who promised to give her the coin later that week in return for the balance of the purchase price. The man took $15,000 and promptly flew out of state with the coin, which he never intended to sell in the first place. With which of the following crimes can the man properly be charged? A. Larceny by trick. B. Embezzlement. C. False pretenses. D. None of the above.

Answer choice C is correct. The crime of false pretenses requires obtaining title to the property (including money) of another person through the reliance of that person on a known false representation of a material past or present fact, when the representation is made with the intent to defraud. Here, the woman transferred title to her $15,000 to the man, relying on his false representations, which were made with the intent to defraud her. Accordingly, the man can properly be charged with the crime of false pretenses. Answer choice A is incorrect. Larceny by trick requires that the defendant obtain possession of, but not title to the property owned by another. Here, the woman intended to pass title to the money, not merely temporary possession, to the man. Accordingly, larceny by trick does not apply.

A bank teller confessed to his colleague that he was concerned about being fired. The colleague suggested that he had similar concerns himself and had been thinking of how he might acquire extra cash in case he lost his job. The teller suggested a plan to take several hundred dollars from their supervisor’s cash drawer, because the supervisor often forgot to lock the drawer when he went to lunch. After the teller took the cash, he would hand it off to his colleague during his lunch break, and the colleague would bring the money to his car to avoid having money on his person at work. The next day, the teller noticed that his supervisor had neglected to lock his cash drawer at lunch, and he carried out the plan. The colleague waited outside the bank, and then took the cash from the teller on his lunch break.

He then hid some of the money in a separate stash he intended to apportion to himself only, without the teller’s knowledge, but he otherwise followed the plan exactly as he and the bank teller had discussed. In a case against the colleague, which of the following charges would most likely be successfully prosecuted? A. Embezzlement and larceny. B. Embezzlement only. C. One charge of larceny only. D. Two charges of larceny only.

Answer choice D is correct. Larceny is the trespassory taking and carrying away of the personal property of another with the intent to permanently deprive that person of the property. An accomplice is a person who, with intent that the crime be committed, aids or abets a principal prior to or during the commission of the crime and is present at the commission of the offense. The accomplice’s assistance to the principal may be verbal encouragement, financial assistance, or physical assistance, provided that the accomplice has the requisite intent to encourage or assist in the commission of the crime. An accomplice is responsible for the crime to the same extent as the principal. In this case, although the colleague did not actually take the money from the supervisor, he aided and abetted the teller in larceny by transferring the money to the car, and thus would be guilty to the same extent as the teller.

The colleague also committed larceny against the teller, as stolen property taken from a thief can constitute larceny. That is, a second thief is guilty of larceny when he steals the property from the first thief, unless the second thief had superior possessory interest in the property. Therefore, answer choice C is incorrect. Answer choice A is incorrect because the teller would not be guilty of embezzlement, which is the fraudulent conversion of the property of another by a person who is in lawful possession of the property. Lower-level employees can generally not be guilty of embezzlement, as such an employee typically has custody, not possession, of the employer’s property. In this case, the teller took the money from his supervisor, who was the one who had possession of the property. Accordingly, he would not be guilty of embezzlement, and thus his colleague would not either. Answer choice B is incorrect because it inaccurately states that the colleague would be guilty of embezzle

After a woman and her roommate got into a fight, the woman and her boyfriend formulated a plan to get back at the roommate. The woman planned to distract the roommate while the boyfriend snuck into the apartment and steal her new camera, which was on the kitchen counter. The woman and her boyfriend headed back to the apartment and put their plan into action. The plan went awry when the roommate heard a noise and went into kitchen to find the boyfriend with her camera in hand. Panicked, the boyfriend pushed the roommate to the floor and ran out of the apartment with the camera. The boyfriend is most likely to be convicted of which of the following crimes? A. Burglary only. B. Burglary and battery. C. Robbery only. D. Robbery and battery.

Answer choice C is correct. Robbery is larceny by force or intimidation, where the taking of the property is from the person or in the presence of the victim. The force need not be great, but must be more than the amount necessary to effectuate taking and carrying away the property. The elements of robbery, including the presence requirement, were satisfied here when the boyfriend pushed down the roommate and ran off with her camera. Answer choices A and B are incorrect because the elements of burglary are not satisfied. Burglary is the breaking and entering of the dwelling of another at nighttime with the specific intent to commit a felony therein. Because the woman was a resident of the apartment, and she let her boyfriend in the apartment, there was no breaking and entering into the dwelling of another. Answer choice D is incorrect because the crime of battery merges into the crime of robbery. Although a battery did occur when the boyfriend pushed the roommate to the floor, the boyf

A husband learned that his wife was having an affair with her boss. The husband was enraged, and offered his brother $50,000 to murder the boss; the brother agreed. The next week, after the husband had given the brother $25,000 as a down payment on the crime, the brother approached the boss in the office parking lot and shot him in the chest. The boss was found shortly thereafter by a colleague, and he was rushed to the hospital but died on the operating table. The husband and his brother were later charged in connection with the boss’s death. The husband is most likely to be convicted of which of the following common-law crimes? A. Murder, conspiracy to commit murder, and solicitation. B. Murder and conspiracy to commit murder only. C. Murder only. D. Solicitation only.

Conspiracy is an agreement between two or more persons to accomplish an unlawful purpose with the intent to accomplish that purpose. The majority rule requires an overt act in furtherance of the conspiracy, although the common law does not. A conspirator can be convicted of both the offense of conspiracy and all substantive crimes committed by any other co-conspirators acting in furtherance of the conspiracy. In this case, the husband and the brother agreed to kill the boss, and the husband committed an overt act—making a payment to his brother—in furtherance of the conspiracy. Thus, the husband would be guilty of both murder and conspiracy. Answer choice A is incorrect because the crime of solicitation “merges” into the completed crime of murder; accordingly, the husband could not be convicted of both solicitation and murder.

The defendant and his brothers decided to rob a bank where the defendant had previously worked. The defendant told his brother everything he knew about the layout of the bank and its security measures. The next day, before the group had taken any further steps to prepare for the robbery, the defendant had a change of heart and told his brothers he would no longer participate or assist them in any way. The brothers put their robbery plans in motion without the defendant and robbed the bank a week later. The security system had changed since the defendant’s employment, however, and the brothers did not notice a teller activate a silent alarm during the robbery. The police captured the brothers as soon as they exited the bank. The defendant has been charged with conspiracy and robbery. The defendant’s sole defense is that he effectively withdrew from the plans for the robbery, and thus cannot be convicted as either a co-conspirator or an accomplice.

The jurisdiction recognizes the majority rule regarding conspiracy and withdrawal from a conspiracy. May the defendant be convicted for either robbery or conspiracy to commit robbery? A. The defendant is likely to be convicted of both robbery and conspiracy to commit robbery. B. The defendant may be convicted of conspiracy to commit robbery, but not of robbery. C. The defendant may be convicted of robbery, but not of conspiracy to commit robbery. D. The defendant is likely to be acquitted of all charges.

Answer choice A is correct. Upon the completion of the overt act, the conspiracy has been formed, and withdrawal is no longer possible. Here, even though the defendant explained that he no longer wanted to be involved with the plan, once he told his brother about the bank's security measures, the conspiracy was complete. A co-conspirator may avoid liability (i.e., may avoid becoming an accomplice) for the underlying crime by repudiating prior aid, doing all that is possible to countermand prior assistance, and doing so before the chain of events is in motion and unstoppable. While the defendant informed his brothers that he no longer wished to participate in the robbery, he did not take any other steps. Accordingly, it was not an effective withdrawal for purposes of accomplice liability; he did not do anything to countermand his prior assistance to his brothers.

Answer choice B is incorrect because the defendant can be convicted of robbery, as an accomplice.An accomplice is a person who, with intent that the crime be committed, aids or abets a principal prior to or during the commission of the crime. The defendant became an accomplice when he gave his brothers the details of the bank's security measures, and did not effectively withdraw from that accomplice liability. Answer choice C is incorrect because the defendant's attempted withdrawal came after the commission of the overt act. The conspiracy was completed when the overt act was committed, and the defendant can therefore no longer withdraw. Answer choice D is incorrect because, as explained, the defendant did not effectively withdraw from conspiracy or the crime itself.

1) The issue is whether Officer could stop the vehicle and arrest an individual based on knowledge acquired through a ___________, rather than through his own personal knowledge. Under the Fourth Amendment right to be free from unreasonable government searches and seizures, a police officer is permitted to stop a vehicle upon a _________________ that detainees are or were involved in criminal activity. This standard requires that there be more than a ______________ but less than probable cause, and need not be based on a police officer’s _________________, Instead, reasonable suspicion can be based on, among other things, a police bulletin.

police bulletin (over the radio) reasonable suspicion vague suspicion personal knowledge

To arrest an individual, a police officer is required to have probable cause that the suspect is committing or has committed a criminal offense. Facts supporting probable cause may come from personal observations, evidence seized during stops based on reasonable suspicion, or evidence discovered in _____________. Any item in public view may be seized without a warrant, because there is no expectation of privacy in things exposed to the public.

plain view

Here, the police officer stopped the car being driven by Bookkeeper based on a police bulletin that advised that a vehicle matching in description and identified by the license plate number of the vehicle driven by Bookkeeper was being driven by an individual suspected of having committed arson. The information supplied by the police bulletin was adequate to develop sufficient and ________________ to stop Bookkeeper’s vehicle.

reasonable suspicion

Upon stopping the vehicle, Officer observed in plain view an empty gas can on the front seat next to the driver. The information obtained through the police bulletin, coupled with the supporting personal observation of the gas can, was sufficient to establish ____________________ to arrest Bookkeeper as a suspect in the arson investigation. The seizure of the gas can was justified because it was observed in Officer’s_______________.

probable cause plain view

Thus, the court was correct in denying defendant Bookkeeper’s motion to suppress the gas can. The officer validly __________ Bookkeeper’s vehicle based on reasonable suspicion obtained from the police bulletin. The information obtained through the police bulletin and Officer’s observation of the gas can justified an arrest based on probable cause, and the seizure of the gas can was permitted as having been observed in plain view.

stopped

(2) The issues are whether Detective’s testimony concerning a deceased witness’s observation violates a defendant’s Sixth Amendment right to confront witnesses against him and whether a ______________ exists to admit the testimony. The Sixth Amendment guarantees an accused the right to encounter adverse witnesses and cross-examine those witnesses in a face-to-face confrontation. Under limited circumstances, face-to-face confrontation is excused when there is an important public-policy reason for doing so, and the reliability of the testimony is otherwise assured, or when the witness’s unavailability is demonstrated to be the defendant’s fault. In addition, the right to confront a witness will be deemed satisfied if defense counsel had a prior opportunity to cross-examine the witness at an earlier hearing. Otherwise, hearsay testimony from a third party is not permitted.

hearsay exception

Additionally, the Sixth Amendment is limited to statements that are _______________ in nature. Out-of-court statements by witnesses that are testimonial are barred under the Confrontation Clause, unless the witnesses are unavailable and the defendant had a prior opportunity to cross-examine those witnesses, regardless of whether such statements are deemed reliable by the court. Out-of-court testimonial statements are not barred by the Confrontation Clause when they are used for a purpose other than establishing the truth of the matter asserted (i.e., when they are not used for a hearsay purpose). A statement is considered testimonial if the declarant would reasonably expect it to be used in a prosecution.

testimonial

Here, the facts indicate that the statements made to Detective by Watchman were not testimonial. The statements were made in the heat of the moment in an attempt to prevent the commission of further crimes and find information to catch the person in the immediate premises, not in preparation for use at trial later. If the __________________ was to save this information for later trial, then it would be a Crawford violation and would be inadmissible. However, here the facts suggest that the statements were nontestimonial and therefore not subject to the Crawford Confrontation Clause issues. Even though Watchman is dead and therefore unable to testify at trial, his statements still could be admitted as evidence because they were not testimonial in natur

motivation

There is also an issue of whether the statements can be considered hearsay. Hearsay is an out-of-court statement offered for the truth of the matter asserted. Hearsay is normally inadmissible, unless an exception applies. There is an exception for an “___________________” when a person, during or soon after perceiving an exciting or intense event, says something about that event. Here, an excited utterance exception may apply. The statement by Watchman was an out-of-court statement that is now being offered to prove that Bookkeeper committed the arson. The arson was an exciting event, and Watchman “exclaimed” his statement; there is no reason to believe that he would be lying right after this event happened, and thus the statement can be admitted under the hearsay exception as well.

excited utterance

(3) The issue is whether the court erroneously prohibited Bookkeeper from discussing her testimony with her attorney during a trial adjournment because she was still under oath. The Sixth Amendment provides a constitutional right to effective assistance of counsel in all felony cases, and it applies at all critical stages of a prosecution. It attaches automatically when a defendant is indicted with formal charges and continues through the sentencing stage of trial. In addition, New York provides accused individuals an indelible right to counsel that offers ________ protection than the Sixth Amendment. This additional protection applies at several stages of a criminal investigation, including trial.

greater

Here, defendant Bookkeeper was in the middle of trial when the court adjourned for the weekend. Bookkeeper was entitled to have the assistance of her counsel through the duration of the trial, including during interim adjournments, and the direction not to discuss the case with her attorney during an adjourned period _______________ her of effective assistance of counsel. Thus, the court’s direction was a violation of defendant Bookkeeper’s Sixth Amendment right to effective assistance of counsel.

deprived

Criminal Law is heavily tested because there are lots of NY distinctions.

be sure to memorize elements of even obvious crimes don't give points away, state the obvious

A. General notes: • Often a stand-alone topic. • Lots of NY _______. Pay attention to New York distinctions from the common law (e.g., burglary in New York has no “at night” component; felony murder falls into two different degrees). Could help to make flashcards or charts for differences

Distinctions

B. Make sure you identify whether it is criminal law or procedure • If criminal law, be sure to articulate the elements. • If criminal procedure, then there are four areas that are tested: 1) 4th amendment search and seizure 2) 5th amendment right against self-incrimination 3) 6th amendment right to counsel 4) Due Process right to fair trial o Mention the Broad Meaning that these stand for.

say where things come from - MIranda comes from the 5th A search warrant stuff comes from 4th A mention the NY distinctions

Due process right to fair trial guarantees, among other things, the right to have a jury consider only evidence admitted at trial. Here, a juror indicated she had visited the scene of the crime and drawn an inference from her visit thereby suggesting information________________ the evidence admitted at trial was considered by at least one juror. This is sufficient to violate D’s due process right to fair trial and justify a mistrial.

other than

Wills - there is almost certainly going to be an essay on this on the bar

-

A police officer suspected that an individual purchased child pornography in violation of the law. The officer sought and secured a facially valid warrant to search the home of the individual for items of child pornography. To obtain the warrant, the officer, in her affidavit, asserted as factual, information received from an informant. The officer executed the warrant and seized as evidence the items specified in the warrant. The prosecution seeks to introduce such evidence against the individual. The individual petitioned the court for a hearing to void the warrant and suppress the evidence seized pursuant to it. The court granted a hearing. At the hearing, the individual established by a preponderance of the evidence that information contained in the affidavit was false, that the officer negligently failed to verify the truthfulness of the information, and that the information was necessary to a finding of probable cause by the magistrate who issued the warrant.

No, because the inclusion of the false information in the affidavit was due to the police officer's negligence. A defendant may go beyond the face of a warrant and challenge its validity due to the inclusion of false information in the affidavit. However, the defendant must establish by a preponderance of the evidence that the false information was knowingly, intentionally, or with reckless regard for its truthfulness included by the affiant in the affidavit and that the information was necessary to a finding of probable cause by the magistrate.

A woman was suspected of murder in the shooting death of her husband and was taken into custody. The interrogating officers did not read the woman her Miranda rights, but instead waited in the room for another officer to arrive. After ninety minutes of waiting, the woman became extremely agitated and blurted out, “I did it! I killed him, and I threw the gun into the river!” The police dragged the river and recovered a gun, and ballistics tests confirmed that the gun was the murder weapon. At trial, the woman’s attorney moved to suppress introduction of both the confession and the gun as evidence. How should the court rule? A. Suppress both the confession and the gun. B. Suppress only the confession. C. Suppress only the gun. D. Suppress neither the confession nor the gun.

D is correct. Any statement obtained as the result of a custodial interrogation may not be used against a suspect at a subsequent trial unless the police first inform the suspect of her Miranda rights. Once a custodial interrogation begins, anything the defendant says is inadmissible until the defendant is informed of the Miranda rights and the defendant knowingly and voluntarily waives those rights. Volunteered statements, however, are not protected by Miranda. Here, the woman’s statements were not made in response to any interrogation on the part of the officers, but rather, were blurted out in a moment of panic before the interrogation began. Therefore, there was no violation of Miranda.

Two officers went to a man’s home to serve an arrest warrant, but no one answered the door. As they walked around the man’s house, the officers looked into his next door neighbor’s window and saw the man inside the neighbor’s kitchen drinking coffee. The officers knocked on the neighbor’s door, and when the neighbor answered, the officers informed the neighbor that they had a warrant to arrest the man. The officers pushed past the neighbor into the kitchen and arrested the man. While they were in the kitchen, the officers saw a bag of marijuana on the neighbor’s counter. The officers arrested the neighbor, and he was subsequently charged with possession of narcotics. The neighbor moved to suppress evidence of the drugs, and the prosecution argued that the evidence was admissible under the plain view exception to the warrant requirement. Are the drugs seized in the neighbor’s kitchen likely to be admitted against the neighbor?

A. No, because the officers could not lawfully enter the neighbor’s home without his consent. B. No, because the officers did not knock and announce to the neighbor their intention to arrest the man. C. Yes, because the arrest warrant implicitly authorized the officers to take measures necessary to serve the warrant. D. Yes, because the officers entered the neighbor’s home under exigent circumstances.

A A warrant to arrest an individual implicitly authorizes entry into the arrestee’s home to serve the warrant if the police have probable cause to believe that the arrestee is present. A police officer may not arrest a person in another person’s home without a search warrant, however, absent exigent circumstances or valid consent from the person whose home is being entered. In this case, the arrest warrant did not authorize the police to arrest the man in his neighbor’s home. Thus, the officers’ presence in the neighbor’s home was unlawful, and any evidence seized therein is inadmissible.

Answer choice B is incorrect because the knock and announce rule pertains to arresting someone in that person’s own home. Even had the police knocked and announced their intention here, the entry would still be unconstitutional if the neighbor had not consented. In any event, even if the knock and announce rule been applicable, failure to knock and announce only invalidates the arrest; it does not trigger the exclusionary rule. Answer choice C is incorrect because the arrest warrant did not authorize the officers to enter the neighbor’s house, regardless of whether it was necessary to serve the warrant. Answer choice D is incorrect because the facts do not indicate that there were exigent circumstances, such as a reasonable belief of danger to the officer or destruction of the evidence.

A defendant sought the services of an out-of-state attorney to represent her against a charge of felony animal abuse. This attorney was erroneously denied pro hac vice admission by the trial court. The defendant then hired an in-state attorney to represent her. The defendant was convicted of the crime, even though the in-state attorney provided the defendant with competent, adequate representation. On appeal, the defendant challenged her conviction on the basis that she was denied her Sixth Amendment right to counsel.

How should the appellate court rule on this challenge? A. Reverse the conviction, because the trial court denied the defendant representation by the counsel of her choice. B. Apply the harmless error standard to determine whether to reverse the defendant's conviction. C. Determine whether there was a reasonable probability that the defendant would not have been convicted if represented by the out-of-state attorney, and if so, reverse the conviction. D. Deny the challenge, because the defendant was provided with competent, adequate representation.

Answer choice A is correct. A defendant is constitutionally entitled to be represented at trial by a qualified attorney of the defendant's choice, where that attorney is not provided by the state. The sanction for violation of this Sixth Amendment right is reversal of the defendant's conviction. Answer choice B is incorrect because the harmless error standard applies to the denial of the right to counsel only in non-trial proceedings.

Answer choice C is incorrect because denial of the right to counsel at trial results in the automatic reversal of the defendant's conviction. The defendant does not need to establish that the defendant was prejudiced by the denial. Answer choice D is incorrect because the defendant need not establish that the replacement counsel's representation was inadequate or that such counsel was incompetent in order to reverse her conviction.

When police go into a house to arrest one person and another person is with them, can the police search the body and clothes of the other person?

Independent justification is needed to search persons not named in a search warrant; mere proximity to a named person does not supply such justification. In this case, the defendant was not named in the search warrant, and the police did not even know his identity. Because the police had no independent justification to search the defendant, the search was illegal, and the gun should be suppressed.

A school district had a strict policy that prohibited the non-medical possession, use, sale or other distribution of any drug on school grounds. Administrators at a local middle school in the district were informed by a student that other students had talked about bringing prescription medications from home to school to take at lunch time for recreational use. One day the following week, the same student gave a school administrator a pill, which the school nurse determined contained prescription strength medication. The student stated that he had been given the pill by another student that morning. Ignoring safety issues, what standard does the administrator need to satisfy in order to conduct a strip search of the other student for possession of prescription medications?

A. None, since a student at a middle school does not have a reasonable expectation of privacy with regard to his body in light of the school district's drug policy and the ease with which prescription medication in pill form can be hidden on his person. B. Reasonable suspicion that the student being searched is hiding prescription medicine in his underwear. C. Probable cause that the student being searched is hiding prescription medicine in his underwear. D. A strip search of student at a public school is per se unreasonable.

Answer choice B is correct. A search conducted by local public school personnel of a student must be based on reasonable suspicion that the search will produce evidence that the student is or has violated school rules. It must also be reasonable in its scope in light of the student's age and gender and the nature of the infraction. This standard applies to strip searches as well as other less intrusive types of searches.

Because a strip search may be allowed under these circumstances, answer choice D is incorrect. Answer choice A is incorrect because a student does have a reasonable expectation of privacy, regardless of the school policy. Answer choice C is incorrect because the school needs only reasonable suspicion, and not probable cause.

On a tip from a criminal informant who indicated that there was "a lot of drug-related" activity going on in a nearby house owned by a woman, a police officer, with no warrant, entered the woman's home and arrested the woman on a charge of criminal drug possession. The officer also seized ten bags of cocaine that were in plain view on the woman's coffee table at the time she was arrested. Which of the following statements regarding these facts is true? A. The bags of cocaine constitute evidence seized pursuant to an unlawful arrest and may be suppressed at trial. B. The woman may not be subsequently prosecuted for the crime of criminal drug possession and if she is, the unlawful arrest will constitute a defense to the crime charged. C. The "plain view" doctrine permits the bags of cocaine to be used in a prosecution of the woman for criminal drug possession. D. The presence of the cocaine following the tip by the informant made the subsequent arrest and seizure of the cocaine lawful.

Answer choice A is correct. Evidence seized pursuant to an unlawful arrest may be suppressed at trial. Here, the woman was arrested in her home, which required a warrant by the police, in the absence of valid consent to enter or exigent circumstances. Answer choice B is incorrect, as an unlawful arrest alone has no bearing on a subsequent criminal prosecution, and is not a defense to the crime charged. Answer choice C is incorrect, as the "plain view" doctrine only applies to evidence seized in a private area if the police officer is legitimately on the premises. Here, the officer had no right to be in the private home and therefore the "plain view" doctrine does not apply. Answer choice D is incorrect. While the tip by the informant may have established probable cause, the police officer did not obtain a warrant.

Where a mistrial is declared despite the defendant's opposition, the defendant cannot be retried unless the mistrial is due to a manifest necessity. A _____________ constitutes a manifest necessity.

hung jury

Statements taken in violation of Miranda may be used to _______________________ of the criminal defendant, if the defendant takes the witness stand and gives testimony at variance with previous admissions. Such statements may not be used as direct evidence, however.

impeach the credibility

A highway was known as a drug corridor used to transport drugs between two cities. The drug unit of a state police force launched an initiative to combat drug trafficking along this section of highway. They formulated a plan to conduct checkpoint stops to check for drugs along certain sections of the highway. Under the plan, the police would stop every hundredth car that passed the checkpoint, and a trained canine would sniff for the presence of drugs. The defendant was stopped at a checkpoint, and the canine detected the presence of cocaine in the trunk of his car. The police then searched the trunk, where they found large amounts of cocaine. The entire stop lasted less than three minutes. The defendant was charged and tried for drug trafficking crimes. At his trial, he moved to suppress evidence of the drugs found during the search. Are the drugs likely to be suppressed?

A. No, because the canine sniff provided probable cause to search the trunk. B. No, because the checkpoint stop was based on neutral, articulable standards. C. Yes, because the checkpoint stop constituted an unreasonable seizure. D. Yes, because a canine sniff of a car may not be performed without probable cause.

Answer choice C is correct. Police may stop an automobile at a checkpoint without reasonable, individualized suspicion of a violation of the law if the stop is based on neutral, articulable standards and its purpose is closely related to an issue affecting automobiles. A roadblock to perform sobriety checks has been upheld, while a similar roadblock to perform drug checks has not. In this case, the checkpoint stops would constitute an unreasonable seizure because their purpose was to only to perform drug checks rather than to prevent an issue affecting automobiles (for example, driving safely).

Answer choice A is incorrect because although the sniff provided probable cause for the search of the trunk, the evidence seized would not be admissible because the stop itself violated the Fourth Amendment. Answer choice B is incorrect because, even if the standards are neutral and articulable, the purpose of the stop must be permissible. Checkpoint stops to perform drug tests are not permissible. Answer choice D is incorrect because the use of a trained dog to sniff for the presence of drugs does not violate the reasonable expectation of privacy present during a valid stop. Thus, a canine search may be performed without probable cause, provided the stop itself meets other constitutional requirements.

An indigent defendant was convicted of violating a state statute prohibiting vagrancy. The defendant unsuccessfully challenged his conviction in a state appellate court on the grounds that the law is unconstitutionally vague on its face. Seeking discretionary review of the appellate court's decision in the state's highest court, the indigent has sought to retain the counsel provided by the state for his trial and initial appeal to prepare this appeal. Is the state constitutionally required to provide the indigent with counsel for this appeal?

A. Yes, because the criminal statute is allegedly unconstitutional on its face. B. Yes, because denial of appointed counsel to an indigent defendant violates the Equal Protection Clause. C. No, because the appeal is discretionary. D. No, because an indigent defendant is not entitled to appointed counsel to pursue any appeal.

Answer choice C is correct. A state is not constitutionally required to provide an indigent defendant with counsel in order to pursue a discretionary appeal. Answer choice A is incorrect because, while an allegation that a statute is unconstitutional on its face can affect whether a person may pursue an appeal, such an allegation is irrelevant to the issue of whether an indigent defendant is entitled to the appointment of counsel.

Answer choice B is incorrect because, although the Equal Protection Clause compels the state to provide an indigent defendant with an attorney for an appeal as of right, this clause does not require the state to provide an attorney for a discretionary appeal. Answer choice D is incorrect because, where a state does provide that a defendant may appeal a conviction as a matter of right, the Equal Protection Clause does compel the state to provide an indigent defendant with an attorney.

An employee of a storage company informed police that the owner of the company was involved in a conspiracy to steal goods and then sell them. According to the employee, the owner permitted the storage of the stolen goods in his warehouse, typically only overnight, before the goods were transported elsewhere for resale. Acting on reliable information from the employee that the warehouse was due to receive a shipment of stolen goods that evening, a police officer immediately sought and obtained a warrant from a neutral and detached magistrate to search the warehouse upon the arrival of the stolen goods. The warrant failed to specify the condition that had to occur before the search was authorized by the warrant. Properly executing the warrant, the police seized the stolen goods. The warehouse owner was charged with conspiracy to commit larceny and possession of stolen goods. The owner sought to suppress the evidence of the stolen goods on the grounds that the seizure was unconstitutiona

Should the court suppress this evidence? A. Yes, because an anticipatory warrant is per se unconstitutional. B. Yes because the failure to state the triggering condition in the warrant caused the warrant to fail for lack of particularity. C. No, because the warrant satisfied the probable cause requirement of the Fourth Amendment. D. No, because warrant is not needed to search business premises such as a warehouse.

Answer choice C is correct. An anticipatory warrant is not unconstitutional simply because the items to be seized are not located on the premises to be searched at the time that the warrant is issued. The probable cause requirement is satisfied where, at the time that the warrant is issued, there is probable cause to believe that the triggering condition will occur and, if that condition does occur, there is a fair probability that contraband or evidence of a crime will be found in a particular place. For this reason, answer choice A is incorrect. Answer choice B is incorrect because a warrant need not state any condition that is precedent to its validity. Answer choice D is incorrect because the warrant requirement generally does apply to the search of a business, particularly where the search is made in regard to criminal activity rather than for administrative purposes.

-

A defendant was charged with theft of a motor vehicle and convicted primarily on an identification of the perpetrator by the individual who purchased the stolen automobile from the defendant. The police did not reveal to the prosecutor that the buyer was serving as a paid police informant with regard to traffic in stolen vehicles. Since the prosecutor was unaware of this information, the prosecutor did not reveal it to the defendant's attorney. The defendant's attorney did not make a discovery request of the prosecutor for evidence favorable to the defendant. Subsequently, upon learning of the buyer's role as a paid police informant, the defendant challenged his conviction on the grounds that the prosecution's failure to reveal such information violated the defendant's due process rights.

Among the following, which is the strongest argument that the state can make to uphold the defendant's conviction? A. The withheld evidence does not exculpate the defendant, but only impeaches the buyer's testimony. B. A prosecutor has no duty to reveal information of which the prosecutor is unaware. C. The defendant made no request for disclosure of such evidence. D. The defendant was not prejudiced by the nondisclosure of the information.

Answer choice D is correct. While, under the Brady rule, a prosecutor has an affirmative duty to disclose any material evidence favorable to the defendant, nondisclosure of such evidence does not violate the defendant's due process rights unless the failure to disclose causes prejudice against the defendant (i.e., that there is a reasonable probability that the defendant's conviction or sentence would have been different had the suppressed evidence been disclosed to the defendant). Since the buyer's identification of the defendant as the perpetrator of the crime was key to the defendant's conviction, the failure to disclose evidence that called into question the validity of that identification could arguably constitute prejudice.

Answer choice A is incorrect because the Brady rule applies to impeachment evidence as well as exculpatory evidence. Answer choice B is incorrect because the Brady rule may apply to evidence of which the police or other state actor is aware, even though the prosecutor is not aware of such evidence. Answer choice C is incorrect because the Brady rule requires the prosecutor to voluntarily provide the defendant with evidence, despite the absence of a request by the defendant for such information, where such evidence is material.

Two police officers stopped a car for a minor traffic violation. While one of the officers dealt with the driver and the traffic violation, the other officer talked with the passenger. Seeking to question the passenger about gang involvement, but lacking reasonable suspicions of criminal activity, the officer ordered the passenger out of the car. As the passenger exited the car, the officer saw a bulge in the passenger's coat which the officer suspected might be a gun. Upon patting down the passenger, the officer felt the handle of a revolver and removed the gun. The passenger, who was a convicted felon, was charged with the possession of a gun by a prohibited possessor. Prior to trial, the defendant sought to suppress the gun as evidence, contending that its seizure was unconstitutional.

Should the court suppress the gun as evidence? A. Yes, because, since the car had been stopped for a traffic violation, the officer could not order the passenger, who had not committed the violation, to exit the car. B. Yes, because, since the officer lacked reasonable suspicion that the passenger was engaged in criminal activity, the officer could not pat down the passenger. C. No, because a valid traffic stop gives an officer the right to pat down a passenger. D. No, because the gun was discovered as a consequence of a valid Terry stop and frisk.

Answer choice D is correct. The traffic violation gave the police officers a valid reason for stopping the car. As part of the stop, the police officer could order the passenger to exit the vehicle. As the passenger did so, the police officer gained a reasonable suspicion that the passenger was armed and dangerous. Consequently, the officer could pat down the passenger for weapons. Answer choice A is incorrect because the police may order a passenger to exit a car that is legitimately stopped for a traffic violation, even though the passenger is not responsible for the traffic violation. Compelling the passenger to exit the car constitutes a de minimis additional intrusion.

Answer choice B is incorrect because the officer's justification for ordering the passenger to exit the car arose from the traffic violation. The fact that the officer also had another reason for doing so that was not supported by a reasonable suspicion as constitutionally required does not prevent the officer from ordering the passenger to exit the car. Answer choice C is incorrect because an officer may not conduct a pat-down as a consequence of a valid traffic stop. The officer must have a reasonable suspicion that the passenger is armed and dangerous.

A city police officer sought and received an anticipatory warrant to search the defendant's premises for evidence of a crime. The defendant, seeking to suppress the evidence seized by the officer during the search, challenged the legality of the warrant. The highest court in the state, while noting that the federal Constitution permits anticipatory search warrants, clearly ruled that the state constitution, which contains language that is similar to the Warrant Clause of the Fourth Amendment, does not. The state has appealed this ruling to the United States Supreme Court.

Which of the following actions must the Supreme Court take? A. Refuse to hear the appeal. B. Reverse the state court decision, because the federal Constitution permits anticipatory warrants. C. Uphold the state court decision, because the federal Constitution does not require anticipatory warrants. D. Uphold the state court decision, because the issue involves a criminal matter, an area traditionally left to the states under the Tenth Amendment.

Answer choice A is correct. The federal Constitution establishes a floor, but not a ceiling, on individual rights. A state is free to interpret its own constitution as granting greater protection to persons than the United States Supreme Court has interpreted the United States Constitution as granting. For this reason, answer choices B and C are incorrect. A final judgment issued by a state court that clearly rests on adequate and independent state grounds is not reviewable by the United States Supreme Court. To do otherwise would require the Supreme Court to issue an advisory opinion. Answer choice D is incorrect because the Tenth Amendment does not prohibit the United States Supreme Court from deciding criminal issues where federal rights are implicated.

-

A defendant is charged as a principal with the rape of a 10-year-old child and with murder, under a felony murder statute, for her unintended death, which occurred during the commission of the rape. Pursuant to state law, the prosecutor timely notifies the defendant of the state's intent to seek the death penalty. The defendant challenges the prosecutor's decision on the grounds that the imposition of the death penalty for these crimes constitutes cruel and unusual punishment under the Eighth Amendment. Should the court permit the prosecutor to seek the death penalty for these crimes?

A. Yes, as to both rape and felony murder. B. No, as to rape, but yes as to felony murder. C. No, as to felony murder, but yes as to rape. D. No, as to both rape and felony murder.

Answer choice B is correct. Capital punishment for the crime of rape, even when the victim is a child, constitutes cruel and unusual punishment under the Eighth Amendment. Accordingly, answer choices A and C are incorrect because they both would allow capital punishment for the crime of rape.

Answer choice D is incorrect because major participation in the felony that resulted in the murder, plus a reckless indifference to human life, is sufficient to satisfy the culpability requirement to justify the death penalty--even absent the intent to kill. Here, although the defendant did not intend the rape victim's death, the defendant committed the violent felony that resulted in her death.

A defendant is charged with theft of services. The state statute provides in relevant part that "a person commits theft of services if the person willingly diverts services that are available only for compensation to the person's own benefit or to the benefit of another in order to avoid payment for the services." The state defines "willingly" as "action with knowledge that the conduct was unlawful." At trial, the defendant admitted that she helped her husband gain access to cable television and internet services in order to avoid payment to the cable provider, and that she knew that such access was unlawful, but asserted that she did so only because he threatened to harm their child unless the defendant helped him. The prosecution requested the court to instruct the jury that the burden of persuasion with regard to issue of whether the defendant acted under duress lies with the defendant. The defense objected to this request, arguing that it was unconstitutional.

How should the court rule on the defense's objection? A. Overrule it, because a state is free to define the elements of a crime as it sees fit. B. Overrule it, because duress does not controvert the elements of theft. C. Grant it, because the defendant, due to duress, did not possess the mens rea necessary to commit theft. D. Grant it, because the defendant is entitled to present evidence in her defense.

Answer choice B is correct. While the prosecution bears the burden of persuasion with regard to each element of a crime, including the necessary intent, the burden of persuasion with regard to an affirmative defense, such as duress, that does not negate the elements of the crime may be placed on the defendant without violation of the Due Process Clause. Answer choice A is incorrect because, although a state is generally free to define the elements of a crime as it sees fit, it may not allocate the burden of persuasion with respect to those elements to the defense without violating the Due Process Clause.

Answer choice C is incorrect because the defendant's alleged duress does not serve to negate the necessary mens rea (i.e., "willingly"--action with knowledge that the conduct was unlawful), which was established by the defendant's own admission, but instead acts as an excuse that, if established, would prevent the imposition of criminal liability for the defendant's otherwise criminal conduct. Answer choice D is incorrect because, while the Due Process Clause does require that a defendant have the opportunity to present evidence, such as evidence of duress, this requirement is irrelevant to the issue of whether, once evidence of duress has been presented, the burden of persuasion with regard to the defense of duress can be placed on the defendant.

A contractor was indicted by a state grand jury for bribery of a public official. The suspect was tried and convicted of the crime. The contractor, who is a member of one racial group, sought to reverse her conviction on grounds that members of another racial group were intentionally excluded from service on the grand jury on the basis of their race. The appellate court found that the exclusion was harmless error with respect to the contractor's conviction.

Should the appellate court reverse the contractor's conviction? A. Yes, because there is a constitutional right to a grand jury indictment. B. Yes, because the members' of the excluded racial group rights under the Equal Protection Clause of the Fourteenth Amendment were violated. C. No, because the exclusion of the racial group was harmless error with regard to the contractor's conviction. D. No, because the contractor lacks standing to challenge the exclusion of members of another race from the grand jury that indicted him.

Answer choice B is correct because a defendant who is indicted by a grand jury from which members of a racial group have been deliberately excluded has standing to raise Equal Protection claims of the excluded racial group, even though the defendant is not a member of the excluded racial group. For this reason, answer choice D is incorrect.

Answer choice A is incorrect because, although the Fifth Amendment requires indictment by a grand jury for federal crimes, this right has not been held to apply to the states by the Supreme Court. Answer choice C is incorrect because, since discrimination in grand jury selection undermines the structural integrity of the judicial process, a conviction obtained as a result of such discrimination is not subject to the harmless error review.

The defendant was the primary suspect in a string of robberies. At the request of the police, the defendant appeared in a line-up at the police station, and one of the victims identified him as the person who robbed him. The police then took the defendant into custody and interrogated him, after which the defendant was released without being charged. The following day, the defendant was arrested and appeared at an arraignment, where he entered a plea of not guilty. After the arraignment, the defendant was appointed an attorney. The defendant had not been represented by an attorney at any point before that time, nor had he requested an attorney. The defendant intends to argue that he was denied his Sixth Amendment right to counsel. At what point did the defendant’s Sixth Amendment right to counsel first attach?

A. The defendant was entitled to counsel at the line-up. B. The defendant was entitled to counsel during the interrogation. C. The defendant was entitled to counsel at the arraignment. D. The defendant was entitled to counsel following the arraignment.

The Sixth Amendment right to counsel generally applies at all critical stages of a prosecution after formal proceedings have begun. The right automatically attaches when the State initiates prosecution with an indictment or formal charge. Here, the right automatically attached when the defendant was arraigned, and he was entitled to the assistance of counsel at the arraignment. Answer choice A is incorrect because pre-charge line-ups are not considered a critical stage of prosecution for the purpose of the Sixth Amendment.

Answer choice B is incorrect because the Sixth Amendment right to counsel does not attach to pre-indictment custodial interrogations. Note, however, that the defendant had the right under the Fifth Amendment to request counsel at that time. Answer choice D is incorrect because the Sixth Amendment guarantees a defendant the right to counsel at an arraignment.

A man was charged with murder. An attorney was appointed due to the man's indigence. The man consulted with his attorney. Subsequently, while the man was imprisoned awaiting trial, the police placed an informant into the man's cell with instructions to elicit incriminating statements about the crime from the man. In response to the informant's prompting, the man confessed to killing the victim. At trial, the man testified that, while present at the scene of the murder, he did not participate in the killing. When the prosecution sought to call the informant as a witness to testify as to the man's prior contradictory confession, the defense attorney objected on the grounds that the confession had been obtained in violation of the defendant's constitutional rights.

Should the court sustain this objection? A. Yes, because the confession was obtained in violation of the defendant's Fifth Amendment right against self-incrimination. B. Yes, because the confession was obtained in violation of the defendant's Sixth Amendment right to counsel. C. No, because the confession was to be used for impeachment purposes. D. No, because the man, as an indigent, was provided with an attorney and was able to consult with his attorney prior to talking with the informant.

Answer choice C is correct. A criminal defendant's Sixth Amendment right to counsel applies once formal proceedings against the defendant with regard to a specific crime have been initiated. Once such proceedings have begun and the defendant has counsel, the police may not seek, either directly or through the use of an informant, to elicit incriminating information from the defendant about that crime without the presence of the defendant's attorney. However, even where a voluntary confession is obtained in violation of a defendant's Sixth Amendment right to counsel, such confession may be used at trial to impeach the defendant's testimony. For this reason, answer choice B is incorrect.

Answer choice A is incorrect because a defendant's Fifth Amendment right against self-incrimination applies to custodial police interrogations. Although the defendant in question was in custody and was being interrogated by the police informant, since the defendant was unaware that the informant was acting at the behest of the police informant, the defendant's confession was not coerced by a police dominated atmosphere and, hence, the defendant's Fifth Amendment right against self-incrimination was not violated. Answer choice D is incorrect because, although the man's indigency did require the state to provide him with an attorney and the opportunity to consult with that attorney, the man's Sixth Amendment right to counsel also required the attorney's presence when the police, acting through the informant, sought to elicit incriminating information from the man.

(1)  The issue is whether Dan committed the crime of larceny against Ann when he forced her by threatening her person and her property to pay back a loan of $900 and she wrote him a bad check to get rid of him.  In New York, extortion is a form of larceny in which property is taken under a threat.  The threat need not be imminent and may include a threat of either physical injury or property damage.  The property intended to be taken need not be on the victim’s person or in the victim’s presence.

Here, Dan obtained the check from Ann under the threat of future physical harm and damage to Ann’s gazebo.  This threat constitutes the crime of extortion, which is a form of larceny.

(2)  The issues are whether Ann committed the crime of issuing a bad check and whether any defenses are available to her.  Under New York law, a person is guilty of issuing a bad check when she issues a check with knowledge that there are insufficient funds to cover the check, believes that payment of the check will be refused, and payment is, in fact, refused.  Full satisfaction of the debt within 10 days of issuing the check is a defense to the crime.

Here, Ann issued Dan the check with knowledge that she did not have sufficient funds to cover the check.  Thus, Ann knew the check would be refused, and, in fact, the check was refused three days later.  Thus, Ann’s actions meet the requisite elements for finding her guilty of the crime of issuing a bad check.


Nonetheless, the facts state that immediately after her check was rejected, Ann deposited sufficient funds to satisfy the check; then, the check was redeposited, and it cleared.  Thus, provided Ann’s deposit of sufficient funds was within the requisite 10 days, she will have a defense to the crime of issuance of a bad check.


Even if Ann’s deposit was not sufficiently timely, she may also plead the affirmative defense of duress.  Duress occurs when a third party’s unlawful threat causes a defendant to reasonably believe that the only way to avoid death or serious bodily injury is to violate the law.  Here, Dan’s threat to Ann that he would hurt her may be sufficient for her to claim that she issued the bad check under duress.

An off-duty police officer was having a drink with his wife in a bar. A man walked by their table, and made a lewd comment about the wife as he passed. The officer jumped to his feet and asked the man to repeat what he had said. The man, who was obviously intoxicated, repeated the comment. The officer then pushed the man, causing the man to fall backwards. The man rose to his feet, pulled a knife from his waistband, and walked toward the officer. As the man lunged at the officer with the knife, the officer pulled his gun from his waistband and shot the man in the leg. Although the officer did not intend to fire a lethal shot, the defendant had a disease that prevented his blood from properly clotting, and he died from blood loss due to the injury. 

Is the officer likely to be convicted of homicide?


A. No, because the officer had no knowledge of the man’s disease when he shot him.B. No, because the officer was entitled to use self-defense under the circumstances.C. Yes, because the officer did not communicate his intent to withdraw from the altercation.D. Yes, because the officer intended to do grievous bodily harm.

B.  An initial aggressor gains the right to act in self-defense where an aggressor using nondeadly force is met with deadly force or the aggressor, in good faith, completely withdraws from the altercation and communicates this fact to the victim. In this case, the officer’s nondeadly force was met with deadly force, and thus he was entitled to use self-defense. 

A woman broke into her ex-husband’s house late one night when she knew he was away on business, intending to take a sculpture that he had been awarded in their divorce settlement. She searched the entire house, but was unable to find the sculpture. She figured he had probably sold the sculpture, which made her furious because he knew how much she loved it. In a rage, she slashed a painting of his new girlfriend before leaving the house.
For which of the following crimes is the woman guilty?


A. Attempted larceny only.B. Larceny only.C. Burglary only.D. Burglary and attempted larceny.

burglary and larceny (larceny does not merge into burglary, only into robbery)

The defendant was upset because he was going to have to close his liquor store due to competition from a discount store in a new shopping mall nearby. In desperation, he decided to set fire to his store to collect the insurance. While looking through the basement for flammable material, he lit a match to read the label on a can. The match burned his finger and, in a reflex action, he dropped the match. It fell into a barrel and ignited some paper. The defendant made no effort to put out the fire but instead left the building. The fire spread and the store was destroyed by fire. The defendant was eventually arrested and indicted for arson. 

The defendant is:


A. guilty, if he could have put out the fire before it spread and did not do so because he wanted the building destroyed.B. guilty, if he was negligent in starting the fire.C. not guilty, because even if he wanted to burn the building there was no concurrence between his mens rea and the act of starting the fire.D. not guilty, because his starting the fire was the result of a reflex action and not a voluntary act.

Answer choice A is correct. Common law arson is the malicious burning of the dwelling of another. Most states have expanded that definition to include the burning of non-dwellings and even buildings owned by the defendant. Even though the defendant wanted to eventually burn down the building, he lacked the malicious intent at the time of the fire. But, if he could have stopped the fire and intentionally did not, that intent would suffice as malicious and he would be guilty of arson. 

When I rob a store and the clerk tries to shoot me and shoots a bystander, am I liable for felony murder on the MBE?

Felony murder is an unintended killing proximately caused by and during the commission or attempted commission of an inherently dangerous felony, including a robbery. Most states apply the proximate cause theory when a bystander is killed by a police officer or due to resistance by the victim of an inherently dangerous felony. Under this theory, the bystander’s death may fall within the felony murder rule because the death is a direct consequence of the felony. In this case, the death of a bystander was a natural and probable consequence of the robbery, and thus proximate causation could be established.

A woman’s husband told her that he was divorcing her, leaving the woman distraught. The woman and her sister decided to burn down the husband’s new house while the husband was at work. The next day, the woman and her sister purchased the necessary supplies and drove to the husband’s house. The sister waited in the car while the woman started the fire. They then drove off to avoid being noticed. Although the woman and her sister believed the husband was at work, in fact he had stayed home sick, and he was killed in the fire. The woman and her sister were both arrested and charged with both arson and homicide.

May the woman and her sister be convicted of both arson and homicide?


A. The woman and her sister may be convicted of arson only.B. The woman and her sister may be convicted of either arson or homicide.C. The woman may be convicted of both arson and homicide, but her sister may be convicted of arson only.D. The woman may be convicted of both arson and homicide, but her sister may not be convicted of either crime.

Answer choice B is correct. The woman could be found guilty of arson because she intentionally burned down her husband's house. Or, she could be convicted of her husband's murder, because the husband's death occurred during the commission of the arson, an inherently dangerous felony. However, because arson will merge with felony murder, she can only be convicted of one or the other. The sister is guilty as an accomplice because she aided and abetted the woman in the commission of the crimes. An accomplice is responsible for the crimes to the same extent as the principal. If the principal commits crimes other than those for which the accomplice has provided encouragement or assistance, an accomplice is liable for the other crimes if the crimes are the natural and probable consequences of the principal's conduct. In this case, the husband's death was a natural and probable consequence of burning down his house. Accordingly, both the woman and her sister would be held liable for either the arson or the husband's death. Answer choice A is incorrect because the woman and her sister would be liable for the husband's death under the felony murder rule, since arson is an inherently dangerous felony. Answer choice C is incorrect because the sister would be held liable for the husband's death since it was a natural and probable consequence of the arson. Answer choice D is incorrect because the sister would be held liable for both crimes as an accomplice.

An employee was up for a promotion but was passed over by his boss for a female colleague with more experience. After learning he had not received the promotion, he became angry with his boss and convinced himself that the colleague and the boss, who was married, were involved in a relationship. He therefore contacted the boss’s wife and convinced her to shoot the boss. They stated that they would not harm the colleague, as it might make their involvement too obvious. The employee provided her with a gun. The next day, the boss’s wife approached the colleague and the boss in their office parking lot. The wife, who was not an experienced shooter, shot the colleague in the arm, and then panicked and ran off. An onlooker rushed the colleague to the hospital. Although the injury was not life threatening, she contracted an infection during surgery and died the following week. A later investigation revealed that the infection was a result of medical malpractice that occurred during the surgery. The crime was eventually traced back to the employee and the boss’s wife, and they were both charged in connection with the colleague’s death. 
The employee is most likely to be convicted of which of the following crimes?


A. Murder, attempted murder, and conspiracy to commit murder.B. Murder, conspiracy to commit murder, and solicitation.C. Attempted murder and conspiracy to commit murder only.D. Murder and conspiracy to commit murder only.

Answer choice A is correct. To prove a homicide, the prosecution must show that the defendant was the actual and proximate cause of the victim’s death. If the victim would not have died but for the defendant’s act, then the defendant’s act is the actual cause of the killing. To prove proximate cause, the death must be foreseeable. A defendant’s conduct is deemed to be foreseeable if death is the natural and probable result of the conduct. Actions by a third party (e.g., medical malpractice by the doctor treating the victim) are generally foreseeable. In this case, the boss’s wife was the actual and proximate cause of the colleague’s death, and thus would be guilty of murder of the colleague. Although the crimes of murder and attempted murder would usually merge, they only merge as to the same person. While the employee could be found guilty of the colleague’s murder, she could also be found guilty of the separate crime of attempting to kill the boss. The employee had the specific intent to kill the boss, and took a substantial step towards that result (approaching the boss in the parking lot and shooting the gun). The employee would be guilty as a co-conspirator. Conspiracy is an agreement between two or more persons to accomplish an unlawful purpose with the intent to accomplish that purpose. A conspirator can be convicted of both the offense of conspiracy and all substantive crimes committed by any other co-conspirators acting in furtherance of the conspiracy. In this case, the employee and the wife agreed to kill the boss, and the employee committed an overt act—providing a gun to the wife—in furtherance of the conspiracy. Thus, the employee would be guilty of both murder and conspiracy, even though the person killed was actually the colleague and not the boss.

A man knew that his uncle regularly withdrew a large amount of cash from a bank and that his uncle was usually not very careful in watching over the money as he made his way home. The man hatched a plan with two friends to steal the money from his uncle. One of the friends purchased a ski mask with which he intended to conceal his face during the heist. A few hours before the event was to occur, the man, deciding to call the whole thing off, told his friends not to go through with the plan. Nevertheless the friends followed the uncle from the bank and successfully absconded with the bag of money without the uncle being the wiser for several minutes. They were subsequently arrested and revealed the man's involvement in the plot. The man was arrested. The applicable jurisdiction has adopted the Model Penal Code. Under the Code, theft is defined as "unlawfully taking, or exercising unlawful control over, movable property of another with purpose to deprive him thereof." The crimes below are listed in descending order of seriousness. 

What is the most serious crime listed for which the man can be convicted?


A. TheftB. ConspiracyC. SolicitationD. No crime

He's guilty of conspiracy and solicitation.  Conspiracy is more serious.  He isn't guilty of the theft, however.  His withdrawal is sufficient to get him out of the theft rap, but not out of the conspiracy rap.  

Is factual impossibility a defense to conspiracy?

no

A statute in the jurisdiction defines murder in the first degree as knowingly killing another person after deliberation. Deliberation is defined as "cool reflection for any length of time, no matter how brief." Murder in the second degree is defined as "all other murder at common law except felony murder." Felony murder is murder in the third degree. Manslaughter is defined by the common law. At 2 a.m., the defendant held up an all-night liquor store using an assault rifle. During the holdup, two police cars with flashing lights drove up in front of the store. In order to create a situation where the police would hesitate to come into the store (and thus give the defendant a chance to escape out the back), the defendant fired several rounds through the front window of the store. The defendant then ran out the back but upon discovering another police car there, surrendered quietly. One of the shots he fired while in the store struck and killed a burglar who was stealing items from a closed store across the street. 

The most serious degree of criminal homicide the defendant is guilty of is:


A. murder in the first degree.B. murder in the second degree.C. murder in the third degree.D. manslaughter.

Answer choice B is correct. Although the defendant could be found guilty of felony murder because another was killed during the commission of an inherently dangerous felony, this fact pattern makes felony murder a lesser crime than second-degree murder. The defendant's actions in firing a gun towards the outside of the store, where the police cars were located, showed a wanton and willful disregard of an unreasonable risk to human life, which amounts to second-degree murder. The defendant cannot be found guilty of murder in the first degree as defined by this statute, because when he fired the gun he did not intend to kill anyone, but rather to create a means of escape. Thus, answer choice A is incorrect. Answer choices C and D are incorrect because murder in the third degree and manslaughter are both less serious crimes than murder in the second degree.

Imperfect self defense = 

 Imperfect self-defense occurs when the person claiming self-defense unjustifiably kills her attacker, such as when she honestly but unreasonably believes self-defense is required. The rule reduces the charge from murder to voluntary manslaughter.

at common law, if I try to get someone to beat up my boyfriend by paying that person $400 and they agree, but then they turn out to be a cop, what am I guilty of?

solicitation and assault (I attempted with the intent to commit the assault by soliciting), but not conspiracy because there is no agreement


 


in NY, there would be conspiracy, too!

Does the majority require an overt act for conspiracy?  Does the common law?  NY?

Yes


No


Yes


 

The defendant, his brother, and his best friend formed a plan to rob a bank. On the day of the crime, the defendant and his brother entered the bank carrying guns, while the best friend stayed in the car to act as a getaway driver. After the defendant received one bag of money from a bank teller, he saw a security guard pull out a gun. The defendant tried to shoot the security guard but instead shot his brother. The defendant panicked and ran out of the building toward the getaway car. The security guard chased the defendant and fired a shot toward him as he approached the car. The shot hit the best friend, who was in the driver’s seat of the car. Shortly thereafter, the police arrived and arrested the defendant. The brother and best friend later died as a result of their gunshot wounds. 
For which of the following crimes is the defendant most likely to be convicted?


A. Robbery and two counts of felony murder for the death of the brother and best friend.B. Robbery and one count of felony murder for the death of the brother.C. Two counts of felony murder for the death of the brother and best friend.D. One count of felony murder for the death of the brother.

Answer choice D is correct. The defendant would be guilty of felony murder for the death of the brother. Felony murder is an unintended killing proximately caused by and during the commission or attempted commission of an inherently dangerous felony, including a robbery. If one of two co-felons kills the other during the commission or attempted commission of a dangerous felony, this will also constitute felony murder. In this case, the defendant killed his brother in the commission of a burglary, and would thus be guilty of murder under the felony murder rule. Answer choice A is incorrect because an underlying felony will generally merge into the crime of felony murder. In this case, the robbery conviction would merge into the felony murder conviction. Answer choice B is incorrect because the defendant would only be liable for one count of felony murder, and the robbery charge would merge into the felony murder charge. Answer choice C is incorrect because, if a co-felon is killed by a victim or a police officer during the commission of an inherently dangerous felony, the defendant is generally not guilty of felony murder.

when you lie to get someone to sell you their home for a very low price, this is false pretenses or larceny by trick?  Or both?

Just false pretenses - larceny by trick means you have possession, but NOT title

I'm an epileptic who has had severe seizures and I voluntarily stop taking my meds and I get in an accident and kill someone.  

This is reckless and criminally negligent, not just negligent (which would be a tort and not a crime)

What is the PInkerton rule?

The “Pinkerton Rule” says that every co-conspirator is guilty of any foreseeable substantive offense committed in furtherance of the conspiracy, regardless of actual knowledge of its commission.

A husband and his friend agreed to kill the husband’s wife, who had been unfaithful, and split the insurance proceeds the man received following the wife’s death. The friend helped the husband obtain a handgun to commit the murder, but on the night of the planned murder, the friend decided that she could not go through with it. Afraid that the husband would kill her if she went to the police, she did not contact them. Instead, she called the husband and told him that she could not go through with the crime and urged him not to kill his wife. The husband ignored her and went through with the killing. The husband and friend were subsequently arrested and the friend was charged with conspiracy to commit murder.

Should the friend be convicted?


A. No, because she successfully withdrew from the conspiracy.B. No, because at the time of the murder, the friend lacked the intent for the wife to be killed.C. Yes, because the friend committed an overt act in furtherance of the conspiracy.D. Yes, because withdrawal is not a valid defense to conspiracy.

Answer choice D is correct. At common law, withdrawal is not a defense to conspiracy because the conspiracy is complete as soon as the parties enter into the agreement. Accordingly, as soon as the man and his friend agreed to kill the man’s wife, the conspiracy was complete. Remember that, unless the fact pattern specifically indicates otherwise, the common-law rule for conspiracy applies on the MBE. Answer choice A is therefore incorrect. Answer choice B is incorrect because conspiracy requires that the conspirator had the intent to agree and the intent to commit the criminal objective at the time of the agreement, not at the time that the criminal objective is accomplished. Answer choice C is incorrect because at common law, no overt act is required for the conspiracy to be complete. All that is required is (i) an agreement, (ii) between two or more persons, (iii) to accomplish an unlawful purpose, (iv) with the intent to accomplish that purpose.

A convenience store clerk was complaining about his financial troubles to his best friend. The friend said that the clerk’s employer had been cheating him out of a decent salary for too many years, and that the employer owed the clerk. The friend suggested that if the clerk robbed the store during another clerk’s shift, he would never get caught. The friend offered to loan the clerk his gun to use to scare the clerk on duty. Both men agreed that no one would get hurt in the process. The next day, the clerk carried out the plan to rob the store while the friend waited outside in the car. During the robbery, the clerk accidentally discharged the gun, and a customer was shot and died instantly. The clerk panicked and left the store empty handed. The friend drove the clerk back to his mother’s house, told him to lay low, and then drove home. The clerk later decided that he needed to get out of town quickly. He stole his mother’s car, which was more reliable than his own, to drive to a nearby state. 
In a case against the friend, which of the following charges would most likely be successfully prosecuted?


A. Attempted robbery, murder, and larceny.B. Felony murder.C. Attempted robbery only.D. Attempted robbery and murder only.

felony murder only - the robbery merges and the larceny didn't involve the friend

A defendant was found guilty of murder. With regard to the separate sentencing phase of the trial to determine whether the death penalty should be imposed, the defendant's attorney did not take the following actions: obtain a social history of the defendant at state expense, which defense attorneys in the state routinely obtained; present witnesses who had testified on behalf of the defendant in the guilt phase due to their ineffective performance during the guilt phase; present mitigating evidence which was subject to an adverse interpretation by the jury; and make a closing argument in the sentencing phase in order to prevent a rebuttal argument by a persuasive prosecutor. 

As described above, which of these actions is most likely to give rise to an ineffective assistance of counsel claim?


A. Failure to obtain defendant's social history.B. Failure to present witness.C. Failure to present mitigating evidence.D. Failure to make a closing argument.

Answer choice A is correct. The first part of the two part test for establishing ineffective assistance of counsel under the Due Process Clause of the Fourteenth Amendment requires that the representation of a defendant by the defendant's attorney fall below an objective standard of reasonableness. There is a presumption of competent representation. An attorney who has failed to utilize an investigative device used routinely by other defense attorneys in the state may be considered not to have met the reasonableness standard. Answer choices B, C, and D each describe a tactical choice made by the defense attorney. Tactical choices rarely fail to meet the standard of reasonableness so long as the attorney does have a tactical reason for making such a choice. Each of these choices reveal such a reason.


 

A police officer obtained a valid warrant to arrest a woman for misdemeanor theft. Having probable cause to believe that the woman was spending the afternoon at a friend's house, the officer went to the friend's house to serve the warrant. No one responded to the officer's knocking or to his identification of himself as a police officer. The officer, finding the door unlocked, opened the door and entered the house. Once in the house, the officer found the woman hiding in a bedroom closet. The woman was properly charged with misdemeanor theft. She sought dismissal of the case against her due the manner of her arrest. 

Should the court dismiss the charges against the woman?


A. No, because the officer arrested the woman pursuant to a valid arrest warrant.B. No, because the woman was properly charged with misdemeanor theft.C. Yes, because the officer's arrest of the woman at her friend's house was illegal.D. Yes, because the officer could not arrest the woman for a misdemeanor unless the misdemeanor occurred in the officer's presence.

Answer choice B is correct. An illegal arrest does not prevent the subsequent prosecution of the person who is illegally arrested. While evidence seized as a consequence of an illegal arrest may be suppressed under the fruit of the poisonous tree doctrine, and the charge may be thrown out if such evidence is necessary for conviction, that is not the case under these facts. Answer choice A is incorrect because, although the woman was arrested pursuant to a valid arrest warrant, the arrest was illegally effected because the officer arrested the woman at the home of her friend. Since the officer did not have a warrant to search for the woman at her friend's home, his arrest of the woman there was illegal. Answer choice C is incorrect because, although the arrest was illegal as noted in answer choice B, the illegal arrest does not prevent the prosecution of the woman for misdemeanor theft. Answer choice D is incorrect because, since the officer had an arrest warrant, he could arrest the woman for a misdemeanor that did not occur in his presence.


 

A defendant was in jail for an aggravated assault charge after his attorney was unable to post bail for that offense. The police believed that the defendant had also been involved with other criminals in executing an unrelated series of bank robberies, and they placed an undercover officer in the jail to pose as the defendant’s cellmate. They hoped to acquire information that might prevent further robberies from occurring. One afternoon, the undercover officer mentioned that he knew the victim of the assault with which the defendant had been charged. The defendant said that the victim had a beating coming to him, and that he was glad he had been the one to complete that task. The next day, the undercover officer mentioned the bank robberies. The defendant began bragging that he had a substantial amount of cash waiting for him once he got out of jail, noting that “all those bankers would hardly miss it.” 
Which of the defendant’s statements to the undercover officer are admissible against him at trial?


A. The statements regarding the assault and the bank robbery are both admissible.B. The statement regarding the assault is admissible, but the statement regarding the robbery is not admissible.C. The statement regarding the robbery is admissible, but the statement regarding the assault is not admissible.D. Neither the statement regarding the assault nor the statement regarding the bank robbery is admissible.

Answer choice C is correct. Once the Sixth Amendment right to counsel is properly invoked, statements that a defendant makes to a police informant are inadmissible when the police intentionally create a situation likely to induce the defendant into making incriminating statements without the assistance of counsel. In this case, the police created a situation likely to induce the defendant into making a statement about the assault without the presence of counsel; that statement would thus be inadmissible. That protection would not apply to the statement about the robbery, however. Under the Sixth Amendment offense-specific standard, the requirement for counsel to be present applies only to interrogations about the offense charged. In this case, the defendant’s Sixth Amendment right to counsel did not apply to questioning about the bank robberies. Moreover, the Fifth Amendment protections are implicated only in the context of a custodial interrogation, and not in the case of questioning by an undercover police officer, as long as the suspect does not know that the interrogator is a police officer. Thus, the statement about the robberies would be admissible. Answer choice A is incorrect because the statement about the assault was taken in violation of the defendant’s Sixth Amendment right to counsel, and it would thus be inadmissible. Answer choice B is incorrect because the statement regarding the robbery did not violate the defendant’s Sixth Amendment right to counsel as related to the robbery. However, he had been charged with assault and had an attorney for that offense, so attempting to induce him to make statements about the assault without his attorney present did violate his rights. Answer choice D is incorrect because the statement about the robbery was not taken in violation of the defendant’s rights and would thus be admissible.

A defendant was convicted of aggravated assault. His attorney appealed the conviction, and the first verdict was overturned based on the judge's finding that after viewing the evidence in the light most favorable to the prosecution, no reasonable fact finder could have voted to convict. The prosecution now seeks to retry the defendant for the same crime. The defendant's attorney objects to the retrial on double jeopardy grounds. 

Should the retrial be allowed?


A. Yes, because a defendant waives any double jeopardy rights by filing an appeal of his conviction.B. Yes, because the Double Jeopardy Clause only prevents retrial when a defendant has been acquitted or when the prosecution dismisses its case.C. No, because retrial after reversal of conviction due to insufficiency of evidence is barred by the Double Jeopardy Clause.D. No, because retrial after a reversal of a conviction on any grounds is prohibited by the Double Jeopardy Clause.

Answer choice C is correct. A conviction based on insufficiency of evidence occurs when, as here, the evidence, when viewed in the light most favorable to the prosecution, would not support a conviction by any reasonable fact finder. When a conviction is reversed on these grounds, jeopardy is implicated, and the prosecution is barred from retrying the defendant for the same offense. Answer choice A is incorrect because, although a defendant who successfully appeals a conviction may generally be retried, a defendant does not waive his double jeopardy rights by filing an appeal. Answer choice B is incorrect because the Double Jeopardy Clause also prohibits retrial of cases overturned due to insufficiency of evidence or if the prosecution asks for a mistrial because of an inability to locate a witness. Answer choice D is incorrect because retrials after appeals on the basis of an error made at trial or after reversal due to weight of evidence are not barred by the Double Jeopardy Clause.

An undercover officer infiltrated a gang by posing as a gang member. While he was working undercover, the officer heard that a member of the gang had shot and killed one of the leaders of a rival gang. The next time the officer and the gang member suspected of the murder were alone, the officer brought up the shooting, indicating that he was very impressed by whoever had the courage to commit such an act. The gang member, taking the bait, bragged that he had shot the rival gang leader, but told the officer to keep the information secret. The gang member was arrested for the murder and the prosecution sought to introduce the statement made to the undercover officer. 
Is the statement likely to be admitted?


A. No, because the gang member was questioned by a police officer without receiving Miranda warnings.B. No, because the officer deceived the gang member as to the officer’s true identity.C. Yes, because the gang member did not know that the undercover officer was a police officer.D. Yes, because the gang member was not subject to interrogation.

Answer choice C is correct. Miranda warnings are not required if the suspect being questioned is not aware that the interrogator is a police officer. Thus, an undercover officer may question a suspect without informing him of his rights, as was done in this case. Note, too, that Miranda warnings only apply to custodial interrogations, and in this case, the gang member was not in custody. Answer choice A is incorrect because the suspect did not know the undercover officer was a police officer, so Miranda warnings were not required. Answer choice B is incorrect because deceit or fraud by an interrogator (e.g., lying about a co-conspirator’s confession) does not itself make a confession involuntary. In this case, the officer’s deception regarding his identity did not make the confession involuntary. Answer choice D is incorrect because interrogation refers not only to express questioning, but also to any words or actions that the police know or should know are likely to elicit an incriminating response. In this case, the officer made a statement that he knew was likely to elicit an incriminating response. While the statement was arguably an interrogation, it was not a custodial interrogation. Accordingly, answer choice C is a better answer.

A mechanic and his former employee were indicted for automobile theft. Unbeknownst to the mechanic, the former employee confessed to the crime and implicated his employer. In exchange for favorable treatment by the prosecutor, the former employee agreed to cooperate in the prosecution of the mechanic. The police were also investigating the mechanic for an alleged plan to kill another person who was to serve as a witness for the state at the mechanic's trial. The mechanic sought a meeting with the former employee. Upon learning of the proposed meeting, the police wired the former employee in order to record the conversation between the mechanic and the former employee. The police directed the former employee to encourage the mechanic to talk about his criminal activity. At the meeting the mechanic made incriminating statements about stealing automobiles. The mechanic's lawyer filed a pretrial motion to suppress these statements on the grounds that his client's right to counsel was violated. 

Should the court grant this motion?


A. No, because the statements were obtained by police during an investigation of a possible crime, a plan to murder a witness, for which the right to counsel had not attached.B. No, because the mechanic initiated the meeting with his former employee.C. Yes, because the police knowingly used the former employee to elicit incriminating statements from the mechanic about the charged crime.D. Yes, because the police used a secret agent to obtain the incriminating statements.

Answer choice C is correct. Since the right to counsel attaches upon the initiation of formal proceedings against the accused, such as indictment, the police cannot seek to elicit incriminating statements from the accused about the crime without the presence of the accused's lawyer unless the accused waives his right to counsel. Seeking such information through a private person who acts as an agent of the police is also prohibited. Answer choice A is incorrect because the fact that the police were also investigating the accused in regard to an uncharged crime for which the right to counsel had not attached does not excuse or justify the police action in seeking to elicit incriminating statements about the charged crime. Answer choice B is incorrect because, even though the accused initiated contact with an unknown police informant, the accused does not waive his right to counsel simply by initiating such contact since the accused is not aware that the informant is acting on behalf of the police. Answer choice D is incorrect because the police are not prohibited from obtaining incriminating statements from the accused through the use of a secret agent so long as the agent does not actively solicit incriminating information from the accused.

In an attempt to combat an increase in drunk driving, police set up a sobriety checkpoint on a street near a strip of nightclubs and bars. The police pulled over every tenth passing vehicle between midnight and 5 a.m. and checked the drivers of these vehicles for signs of intoxication. As part of this process, an officer pulled over the defendant’s car, although the defendant was not exhibiting signs of intoxication. When the defendant opened his window, the officer detected a very strong smell of marijuana. The officer ordered the defendant out of the vehicle and searched the defendant’s car. The officer found a baggie containing marijuana under the floor mats in the back seat of the car. The defendant was arrested and charged with possession of narcotics. The defendant seeks to suppress evidence of the marijuana.

Is the defendant likely to succeed?


A. No, because the search was incident to a lawful arrest.B. No, because the search was permitted based on the automobile exception to the warrant requirement.C. Yes, because the officer did not have reasonable suspicion to stop the defendant, and thus any fruits of the defendant’s seizure are inadmissible.D. Yes, because no exception to the warrant requirement permitted the officer to search the back seat of the defendant’s car.

Answer choice B is correct. The Fourth Amendment does not require police to obtain a warrant to search a vehicle if they have probable cause to believe it contains contraband or evidence of a criminal activity. In this case, the strong smell of marijuana coming from the car provided probable cause to believe that the car might contain contraband, and thus the officer could search the car without first obtaining a warrant. Answer choice A is incorrect because the arrest was not made until after the search was completed. Answer choice C is incorrect because police may organize checkpoint stops if the stops are based on neutral articulable standards and their purpose is closely related to an issue affecting automobiles, such as drunk driving. Accordingly, the police were permitted to stop the defendant as part of this sobriety checkpoint. Answer choice D is incorrect because the automobile exception to the warrant requirement allows police to search anywhere in the car where they believe there to be contraband, including the trunk, as long as they have probable cause to do so.


 

After a defendant was arrested and charged with robbery but before he had an attorney, the police brought him in to question him about the robbery. After being read his Miranda warnings, the defendant said nothing. For the next hour, the police questioned the defendant, and he continued to remain silent. Finally, one officer mentioned that the defendant’s mother was going to be miserable sitting through his trial, and the defendant broke down. He admitted that he had been the driver for the robbery, but that it had been a mistake, and he did not want to make his mother sit through a trial. The defendant later sought to suppress his statement. 
Is the defendant’s statement likely to be suppressed?


A. No, because the defendant never specifically invoked his right to counsel.B. No, because the defendant waived his Miranda rights when he spoke to police.C. Yes, because the circumstances indicate that the defendant’s statement was not voluntary.D. Yes, because the defendant was entitled to the presence of counsel.

Answer choice D is correct. The Sixth Amendment right to counsel applies at all critical stages of a prosecution, after formal proceedings have begun. The right automatically attaches when the State initiates prosecution with an indictment or formal charge and ends at the sentencing stage of the trial. A critical stage includes a custodial interrogation. In this case, the defendant’s Sixth Amendment right to counsel had attached, and thus he was entitled to the presence of counsel during the interrogation. He did not need to formally invoke this right for it to attach. Because his statement was taken in violation of his Sixth Amendment right, it would be suppressed. Answer choice A is incorrect because although a defendant is required to specifically invoke his right to counsel under the Fifth Amendment, there is no such requirement under the Sixth Amendment. Thus, the defendant was entitled to counsel even if he did not make a specific request. Answer choice B is incorrect because although the defendant waived his right to remain silent by making a statement, he did not waive his right to counsel. His statement was taken in violation of his Sixth Amendment right to counsel and would thus be suppressed. Answer choice C is incorrect because the facts do not indicate that the statement was involuntary. The police officer’s statement about the defendant’s mother does not elevate the circumstances to a level of making the statement involuntary.

Two undercover police officers, with probable cause to believe that the defendant was a drug dealer, entered the living room of the defendant's apartment, at the defendant's invitation, to buy cocaine. Before the transaction could take place, the defendant shot and killed one of the officers. After a brief struggle, the defendant was subdued by the other officer and placed under arrest for murder. Responding to the officer's request for assistance, uniformed police officers came to the apartment, conducted a protective sweep, and took the defendant to jail. Then the uniformed officers conducted a thorough warrantless search of the apartment, during which they uncovered a large quantity of cocaine in the mattress in the defendant's bedroom. Based on the amount of cocaine seized, the defendant was charged with possession of cocaine with intent to deal in addition to murder. The defendant filed a motion to suppress the cocaine as having been unconstitutionally seized. 

Should the court grant this motion?


A. Yes, because the search was conducted without a warrant.B. Yes, because the defendant was arrested for murder not drug dealing.C. No, because the defendant had a lesser expectation of privacy once arrested.D. No, because the murder created exigent circumstances.

Answer choice A is correct. A search of a suspect's home without a warrant is per se unreasonable unless a warrant exception, such as consent or exigent circumstances, applies. Although the undercover officers presumably did enter the defendant's apartment with the defendant's consent, that consent extended only to the living room, the area in which the cocaine purchase was to have taken place. Nothing that the defendant did subsequently would indicate an intent to extend the area of that consent. As for exigent circumstances, although the murder certainly justified the protective sweep of the apartment, the arrest and removal of the defendant eliminated any emergency that would have prevented the police from obtaining a warrant before searching the apartment for evidence related to the murder or drug dealing. Consequently, answer choice D is incorrect. Answer choice B is incorrect because, even though the defendant was arrested for murder, police may search for evidence of any crime for which they have probable cause, provided the warrant requirement is met. The suspect need not be arrested prior to a search for evidence of the crime. (Note: The existence of drugs in the defendant's apartment could constitute evidence of motive with regard to the murder of the police officer, as well as evidence directly related to the drug dealing charge.) Answer choice C is incorrect because, although the arrest of a suspect in his home does lessen the expectation of privacy in the suspect's person, it does not alter the suspect's expectation of privacy with regard to his home, beyond the invasion occasioned by a protective sweep.


 

A defendant was visiting his girlfriend’s home. Before leaving, he hid a knife in the kitchen cabinet without her knowledge. Later that day, the police arrived at the girlfriend’s house to question her about a recent stabbing in which the defendant was a suspect. The police believed the girlfriend was withholding information and threatened to arrest her for obstruction of justice if she did not allow them to search her home. The girlfriend consented, and the police discovered the knife in the kitchen. The knife was later identified as the weapon used in the stabbing, and the defendant was charged and tried for the crime. The defendant moved to suppress the knife, and the prosecution objected. 
Is the knife likely to be admitted against the defendant at trial?


A. No, because the girlfriend’s consent to the search was not valid against the defendant.B. No, because the girlfriend did not provide voluntary consent for the search.C. Yes, because the girlfriend consented to the search.D. Yes, because the search did not violate the defendant’s reasonable expectation of privacy.

Answer choice D is correct. Only unreasonable searches and seizures are subject to the Fourth Amendment. A search occurs when governmental conduct violates a reasonable expectation of privacy. An individual generally does not have a reasonable expectation of privacy in the home of another in which the individual was merely a visitor (although an overnight guest may have a reasonable expectation of privacy). In this case, the defendant did not have a reasonable expectation of privacy in his girlfriend’s home, and thus his rights were not violated by the search. Accordingly, the knife would be admissible. Answer choice A is incorrect because the issue is not whether the girlfriend’s consent was effective as related to the defendant but instead whether the defendant had an expectation of privacy in his girlfriend’s home. The defendant had no expectation of privacy in his girlfriend’s home, and thus the issue of consent is not relevant in determining whether the evidence may be admitted against the defendant. Answer choice B is incorrect because, although the consent was likely not voluntary given the threats made by the police, the defendant’s rights were not violated because he had no standing to raise this challenge. Thus, although the girlfriend might be able to suppress evidence from the search used against her, the defendant would have no such right. Answer choice C is incorrect because it is not relevant whether there was consent for the search or whether the search itself was legal. The search did not violate the rights of the defendant, and thus he may not suppress evidence found during the search.


 

Answer choice D is correct. Only unreasonable searches and seizures are subject to the Fourth Amendment. A search occurs when governmental conduct violates a reasonable expectation of privacy. An individual generally does not have a reasonable expectation of privacy in the home of another in which the individual was merely a visitor (although an overnight guest may have a reasonable expectation of privacy). In this case, the defendant did not have a reasonable expectation of privacy in his girlfriend’s home, and thus his rights were not violated by the search. Accordingly, the knife would be admissible. Answer choice A is incorrect because the issue is not whether the girlfriend’s consent was effective as related to the defendant but instead whether the defendant had an expectation of privacy in his girlfriend’s home. The defendant had no expectation of privacy in his girlfriend’s home, and thus the issue of consent is not relevant in determining whether the evidence may be admitted against the defendant. Answer choice B is incorrect because, although the consent was likely not voluntary given the threats made by the police, the defendant’s rights were not violated because he had no standing to raise this challenge. Thus, although the girlfriend might be able to suppress evidence from the search used against her, the defendant would have no such right. Answer choice C is incorrect because it is not relevant whether there was consent for the search or whether the search itself was legal. The search did not violate the rights of the defendant, and thus he may not suppress evidence found during the search.


 

Answer choice B is correct. There is a right to a jury trial for offenses that carry an authorized sentence of more than six months, regardless of the actual penalty imposed. In this case, the maximum sentence for each offense was only six months, and thus the right to a jury trial was not triggered. Answer choice A is incorrect because the actual sentence imposed is irrelevant in determining whether the right to a trial by jury attaches; rather, the right hinges on the maximum authorized sentence. Answer choice C is incorrect because the maximum prison term for each offense cannot be aggregated. Thus, there is no right to a trial by jury for multiple petty offenses that carry a combined total maximum term exceeding six months. Answer choice D is incorrect because a maximum fine of $5,000 in addition to six months imprisonment does not convert the crime into a serious offense.

A defendant was tried for theft of a motor vehicle and found guilty. As permitted by state law as a matter of right, he sought to appeal his conviction. His attorney, whom the defendant had hired, timely filed a notice of appeal. However, the appellate court dismissed the appeal due to his attorney's failure to file the necessary supporting documentation required by state law. The defendant subsequently challenged the action of the appellate court on the grounds of ineffective assistance of counsel. 

Will the defendant's challenge likely be successful?


A. No, because the attorney timely filed a notice of appeal.B. No, because there is a presumption that an attorney's representation of a client was constitutionally adequate.C. No, because the protection against ineffective assistance of counsel only applies to state-provided counsel.D. Yes, because the attorney's unreasonable conduct prevented the defendant from pursuing his appeal.

Answer choice D is correct. There is a two-part test for establishing ineffective assistance of counsel: (i) the representation of a defendant by the defendant's attorney must fall below an objective standard of reasonableness, and (ii) the attorney's deficient performance prejudiced the defendant. Here, although the attorney took the necessary first step of filing a notice of appeal, the attorney unreasonably failed to take the second step of filing the necessary document. As a consequence, the defendant was prejudiced (i.e., his appeal was dismissed without a hearing). Answer choice A is incorrect because, although the defendant's attorney did timely file a notice of appeal, the attorney's failure to file the documentation necessary to pursue the appeal was unreasonable. Answer choice B is incorrect because, although there is a presumption that an attorney's representation of a client was constitutionally adequate, this presumption may be overcome. Answer choice C is incorrect because the protection against ineffective assistance of counsel extends to an attorney hired by a defendant as well as to court-appointed counsel.

Under what circumstance is it a violation of the 8th Amendment to sentence a minor to life imprisonment without the possiblity of parole?

 A sentence of life imprisonment without the possibility of parole constitutes "cruel and unusual punishment" under the Eighth Amendment when the sentence is imposed for a non-homicide crime on a defendant who was a minor at the time that the crime was committed. 

A defendant was on trial for arson of an office building. During the trial, the police officers who initially arrested him began to suspect that the defendant was also responsible for a theft of several computers that took place in the office building on the same night as the arson. One evening during the arson trial, the police visited the defendant in jail, read him his Miranda rights, and began to question him about the theft. The defendant eventually confessed, and the police informed the prosecutor of his confession. The prosecutor later wants to charge the defendant with theft, using the statement that was obtained by the police in the jail. The defendant's attorney objects on the grounds that the defendant's Sixth Amendment rights were violated. 

The court should:


A. Allow the prosecutor to use the confession because the defendant was read his Miranda rights.B. Allow the prosecutor to use the confession because theft and arson each require proof of an additional element that the other does not.C. Prevent the prosecutor from using the confession because the arson and the theft took place during the same criminal transaction.D. Prevent the prosecutor from using the confession because the defendant was represented by counsel in connection with the arson case.

Answer choice B is correct. The Sixth Amendment is offense-specific, meaning that the interrogation that is the subject of the Sixth Amendment inquiry must relate to the crime for which criminal proceedings have commenced. The Sixth Amendment right to counsel does not attach to other crimes for which the accused may be under investigation but which are unrelated to the pending prosecution. Under Blockburger v. United States, 284 U.S.299 (1932), two different crimes committed in one criminal transaction are deemed to be the same offense for Sixth Amendment purposes unless one offense requires proof of an element that the other does not. Since arson and theft each require elements the other crime does not, they are not the same crime for Sixth Amendment purposes and the confession may therefore be used to charge the additional crime. Answer choice A is incorrect because, while a Miranda warning was necessary in order to elicit an admissible confession, Miranda warnings have no effect on a defendant's Sixth Amendment rights. Answer choice C is incorrect because, as discussed above, two different crimes in one criminal transaction are not the same for Sixth Amendment purposes if one requires proof of an element that the other does not. Answer choice D is incorrect because the Sixth Amendment right to counsel is offense-specific; it applies only to the offense at issue in those proceedings.


 

An officer pulled over the defendant for speeding. When he ran the defendant’s plates, the officer saw that there was an outstanding warrant for the defendant’s arrest based on her failure to pay child support. The officer arrested the defendant and placed her in the backseat of his squad car. The officer then returned to the defendant’s car and saw that her purse was sitting on the passenger seat. The officer searched the purse and found a small amount of marijuana. The defendant was later charged with drug possession. She has moved to suppress evidence of the marijuana. 
Is the defendant likely to succeed in having evidence of the marijuana suppressed?


A. No, because the officer was permitted to conduct a search of containers immediately associated with the defendant, including her purse, incident to a valid arrest.B. No, because the officer was permitted to conduct a search of the passenger compartment of the defendant’s vehicle incident to a valid arrest.C. Yes, because the officer did not have a reasonable belief that the passenger compartment of the vehicle contained evidence of the offense.D. Yes, because the officer could not search the vehicle without probable cause to believe that the vehicle contained contraband.

Answer choice C is correct. In order to justify a warrantless search of an automobile incident to arrest, the Fourth Amendment requires that law enforcement demonstrate either (i) that the arrestee is within reaching distance of the passenger compartment at the time of the search and, as a result, may pose an actual and continuing threat to the officer’s safety or a need to preserve evidence from being tampered with by the arrestee or (ii) that it is reasonable that evidence of the offense of arrest might be found in the vehicle. In this case, the defendant was not within reaching distance of the passenger compartment of the car, and it was not reasonable that evidence of the violation of a child support order would be found in the car. Accordingly, the officer could not search the purse without a warrant. Answer choice A is incorrect because, although an officer may search an individual and containers “immediately associated” with the individual pursuant to a lawful arrest, in this case the purse was in the vehicle and not immediately associated with the defendant. Answer choice B is incorrect because, although the Belton rule previously provided that an officer could conduct a full search of a vehicle pursuant to a lawful arrest, the Supreme Court has since limited the parameters of such a search as previously described. Answer choice D is incorrect because an officer is not necessarily required to have probable cause to search a vehicle incident to a lawful arrest. Accordingly, answer choice C is a better answer.

A defendant entered a guilty plea in state court for conduct stemming from a single incident in which the defendant broke into the victim's apartment with the intent of committing a rape and did so. The judge entered a judgment against the defendant on both the burglary and rape charges. Pursuant to state law, sentences for two or more crimes must run concurrently unless the judge finds that the crime was an indication of the defendant's willingness to commit more than one crime. The judge, making such a finding with respect to burglary, ordered that the sentences for burglary and rape run consecutively. The defendant filed an appeal, challenging the consecutive sentences as unconstitutional. 

Should the court rule in the defendant's favor?


A. No, because a judge may determine whether a defendant's sentences run consecutively.B. No, because, where a defendant enters a guilty plea, the defendant has waived his right to a trial by jury, and the judge may find a fact that results in the enhancement of the defendant's sentence.C. Yes, because the Double Jeopardy Clause of the Fifth Amendment as applied to the states through the Fourteenth Amendment prohibits punishment for both a crime and its lesser included offense.D. Yes, because a jury, not a judge, must find any fact essential to a defendant's punishment.

Answer choice A is correct. A judge may make the determination of whether sentences for separate crimes may run consecutively. The Sixth Amendment right to a jury trial does not require that a jury, rather than a judge, find any fact necessary to impose consecutive, rather than concurrent sentences. Answer choice B is incorrect because, even where a defendant enters a guilty plea, a jury, rather than the judge, must find a fact that results in the enhancement of the defendant's sentence with respect to a specific offense. However, where the defendant is convicted of multiple offenses, the judge may determine whether the sentences for each offense run consecutively or concurrently. Answer choice C is incorrect because, although the Double Jeopardy Clause does prohibit punishment for both a crime and its lesser included offense, rape is not a lesser included offense of burglary. Although burglary typically requires breaking and entering of a dwelling with the intent to commit a felony, which in this instance is rape, it does not require proof that the defendant committed a rape. Consequently, rape is not a lesser included offense of burglary. Answer choice D is incorrect because, while the Sixth Amendment right to a jury trial does require that a jury, not a judge, find any fact essential to a defendant's punishment, it is limited to the determination of discrete offenses and the sentences for each. It does not extend to the determination of whether the sentences for such offenses should run concurrently or consecutively.


 

Police executed a valid warrant to search for heroin in the defendant's residence. Finding the defendant at home, the police detained him in handcuffs for the duration of the search. The police found a small amount of heroin in the defendant's bedroom during the search. Upon the completion of the search, the police arrested the defendant for possession of heroin. The police then searched the defendant's person and found a larger quantity of cocaine. At his trial for possession of cocaine, the defendant sought to suppress the cocaine as having been unconstitutionally seized from his person. 

Should the court suppress the cocaine?


A. Yes, because the handcuffing of defendant constituted a per se unreasonable seizure.B. Yes, because the police lacked probable cause to arrest the defendant for possession of cocaine at the time of his detention.C. No, because the police were authorized to seize the cocaine by a search warrant.D. No, because the cocaine was found pursuant to a search incident to a valid arrest.

Answer choice D is correct. The cocaine was found during a search of the defendant's person after the defendant was arrested for possession of heroin. This arrest was valid because the heroin was found during the execution of a warrant which authorized the search for it. Answer choice A is incorrect because, while a defendant's detention during the execution of a search warrant does constitute a seizure, a defendant's detention by handcuffing is not per se unreasonable. Answer choice B is incorrect because, although the police did lack probable cause to arrest the defendant for possession of cocaine at the time of his detention, the police could detain the defendant during the execution of the search warrant. Upon finding the heroin, the police were justified in arresting the defendant. Having arrested the defendant, the police could constitutionally perform a search incident to the arrest, during which search the cocaine was discovered. Answer choice C is incorrect because, although the police had a valid search warrant, the warrant authorized a search for heroin, not cocaine.

A customer and her companion went into a drugstore, where the customer reached into the cash register and took out $200. The owner of the store came out of a back room, saw what had happened, and told the customer to put the money back. Her companion then took a revolver from under his coat and shot and killed the store owner. The customer claims that the store owner owed her $200 and that she went to the drugstore to try to collect the debt. She said that she asked her companion to come along just in case the store owner made trouble but that she did not plan on using any force and did not know that her companion was armed. 

If the customer is prosecuted for murder on the basis of felony murder and the jury believes her claim, she should be found:


A. guilty, because her companion committed a homicide in the course of a felony.B. guilty, because the customer taking her companion with her to the store created the risk of death that occurred during the commission of a felony.C. not guilty, because she did not know that the companion was armed and thus did not have the required mental state for felony murder.D. not guilty, because she believed she was entitled to the money and thus did not intend to steal.

Answer choice D is correct. Felony murder is murder committed during the perpetration of a felony that is considered dangerous to life, such as robbery. Robbery is a larceny from a person by force or intimidation. Here, the customer has a valid defense to robbery. If the customer did not intend to steal the money, she could not complete the element of larceny that requires an intent to steal at the time of the taking. Accordingly, if the customer is not guilty of a felony, she cannot be guilty of felony murder.

The defendant joined a neighborhood gang. At a gang meeting, as part of the initiation process, the leader ordered the defendant to kill a member of a rival gang. The defendant refused, saying he no longer wanted to be part of the group. The leader, with the approval of the other members, told the defendant that he had become too involved with the gang to quit and that they would kill him if he did not accomplish the murder of the rival gang member. The next day, the defendant shot a rival gang member to death while he was sitting on his motorcycle outside a restaurant. The defendant is charged with first-degree murder. First-degree murder is defined in the jurisdiction as the intentional premeditated killing of another. Second-degree murder is all other murder at common law. 

If the defendant killed the rival gang member because of the threat to his own life, the defendant should be found:


A. not guilty, because of the defense of duress.B. not guilty, because of the defense of necessity.C. guilty of first-degree murder.D. guilty of second-degree murder.

Answer choice C is correct. The defendant is guilty of first-degree murder because he killed the rival gang member intentionally and with premeditation. Even though the defendant was being coerced by the gang to commit the crime, he does not have a valid duress or necessity defense because the threat was not immediate or imminent; the defendant had other means of escape, and was arguably at fault for exposing himself to such a threat by attending a gang initiation meeting. Also, claims of duress or necessity are inapplicable when the crime committed is homicide, so answer choices A and B are incorrect. Answer choice D is incorrect because the defendant's killing of the rival gang member fell within the provided definition of first-degree murder.

After waiting until all the customers had left, a man entered a small grocery store just before closing time. He went up to the only clerk in the store and said, "Hand over all the money in the cash register or you will get hurt." The clerk fainted and struck his head on the edge of the counter. As the man went behind the counter to open the cash register, two customers entered the store. The man ran out before he was able to open the register drawer. 

On this evidence, the man could be convicted of:


A. robbery.B. assault and robbery.C. attempted robbery.D. assault and attempted robbery.

Answer choice C is correct. Robbery is larceny from the person or their immediate presence by force or intimidation. An attempt is a substantial step towards the commission of a crime coupled with the intent to commit the crime. Here, the man entered the store with the intent to steal money (attempt to commit robbery) and threatened the clerk. However, the man failed to complete the crime of robbery (and answer choice A is incorrect) because he did not take or carry away the money. Accordingly, he is guilty of attempted robbery. Answer choices B and D are incorrect because, as a lesser-included offense, the assault charge merged into the attempted robbery charge.

A father in desperate need of money decided to hold up a local convenience store. Determined not to harm anyone, he carried a toy gun that resembled a real gun. In the store, he pointed the toy gun at the clerk and demanded money. A customer who entered the store and saw the robbery in progress pulled his own gun and fired at the father. The bullet missed the father but struck and killed the clerk. The father was charged with felony murder. 

His best argument for being found NOT guilty is that he:


A. did not intend to kill.B. did not commit the robbery because he never acquired any money from the clerk.C. did not intend to create any risk of harm.D. is not responsible for the acts of the customer

Answer choice D is correct. Of the answer choices, only answer choice D, if true, would help the father. His best argument is that because he did not have a real weapon, the murder was the result of an unforeseeable, intervening act for which he is not responsible. Answer choice B is incorrect because a felony does not have to be complete for the felony murder rule to apply. Answer choices A and C are incorrect because the father's intent does not alleviate him from liability for the crime under the felony murder rule.

A bank teller was fired by the president of the bank. The bank teller decided to take revenge against the bank president, but decided against attempting it personally, because he knew the bank president was protected around the clock by bank security guards. The bank teller knew that his friend had a violent temper and was very jealous. The bank teller falsely told his friend that his wife was having an affair with the bank president. Enraged, his friend said, "What am I going to do?" The bank teller said, "If it were my wife, I'd just march into his office and blow his brains out." The friend grabbed a revolver and rushed to the bank. He walked into the bank, carrying the gun in his hand. One of the security guards, believing a holdup was about to occur, shot and killed the friend. 

If charged with the murder of his friend, the bank teller should be found:


A. guilty, based upon extreme recklessness.B. guilty, based upon transferred intent.C. not guilty, because he did not intend for his friend to be shot by the security guard.D. not guilty, because he did not shoot his friend and he was not acting in concert with the security guard.

Answer choice A is correct. Murder is the unlawful killing of a human being committed with malice aforethought. When the bank teller purposefully inflamed his friend in an effort to have the bank president killed, his extreme indifference to the value of human life (depraved heart) resulted in his friend's death. Answer choice B is incorrect. The doctrine of transferred intent is inapplicable here, because the bank teller did not kill his friend in his attempt to actually kill the bank president. Instead, the bank teller was reckless with his friend's life. Answer choice C is incorrect because the bank teller need not have intended to kill his friend to have the requisite intent under murder; extreme recklessness suffices. Although the statements in answer choice D are correct, they are not the only scenarios in which the bank teller could be found guilty of murder.

A woman drove her car through the drive-through lane of a fast-food restaurant in the afternoon. When she reached the microphone used to place orders, she said, "There's a man across the street with a rifle. He can see everything you do. If you do not do exactly what I tell you, he will shoot you. Put all the money from the register into a sack and give it to me when I drive up." The clerk did not see anyone across the street and was unsure whether anyone was there. However, unwilling to risk harm to himself, he put $500 in a paper bag and handed it to the woman when she drove up to the delivery window. The woman drove off with the money but was arrested a short time later. She had lied about the man with a rifle and had acted alone. 

Of what crime or crimes can the woman be convicted?


A. Embezzlement.B. Obtaining property by false pretenses.C. Robbery and larceny.D. Robbery or larceny.

Answer choice D is correct. All the elements of larceny and robbery (which is larceny plus force or intimidation) were present. The woman's threat of immediate harm to the clerk was sufficient to constitute the intimidation required for robbery. Answer choice A is incorrect because the woman never had lawful possession of the money and so cannot be convicted of embezzlement. Answer choice B is incorrect as the crime was not false pretenses because, among other things, the woman never obtained title to the cash. Answer choice C is incorrect because larceny is a lesser included offense of robbery. A defendant cannot be convicted of both for a single offense.